NURS 4000 Exam 1

Pataasin ang iyong marka sa homework at exams ngayon gamit ang Quizwiz!

When planning emergent care for a patient with a suspected MI, what should the nurse anticipate administrating? A Oxygen, nitroglycerin, aspirin, and morphine B Oxygen, furosemide (Lasix), nitroglycerin, and meperidine C Aspirin, nitroprusside (Nipride), dopamine (Intropin), and oxygen D Nitroglycerin, lorazepam (Ativan), oxygen, and warfarin (Coumadin)

A Oxygen, nitroglycerin, aspirin, and morphine The American Heart Association's guidelines for emergency care of the patient with chest pain include the administration of oxygen, nitroglycerin, aspirin, and morphine. These interventions serve to relieve chest pain, improve oxygenation, decrease myocardial workload, and prevent further platelet aggregation. The other medications may be used later in the patient's treatment.

When providing nutritional counseling for patients at risk for CAD, which foods would the nurse encourage patients to include in their diet (select all that apply)? A Tofu B Walnuts C Tuna fish D Whole milk E Orange juice

A Tofu B Walnuts C Tuna fish Tuna fish, tofu, and walnuts are all rich in omega-3 fatty acids, which have been shown to reduce the risks associated with CAD when consumed regularly.

The community health nurse is planning health promotion teaching targeted at preventing coronary artery disease (CAD). Which ethnic group would the nurse select as the highest priority for this intervention? A White male B Hispanic male C African American male D Native American female

A White male The incidence of CAD and myocardial infarction (MI) is highest among white, middle-aged men. Hispanic individuals have lower rates of CAD than non-Hispanic whites or African Americans. African Americans have an earlier age of onset and more severe CAD than whites and more than twice the mortality rate of whites of the same age. Native Americans have increased mortality in less than 35-year-olds and have major modifiable risk factors such as diabetes.

Which action will the nurse include in the plan of care for a 72-year-old woman admitted with multiple myeloma? a. Monitor fluid intake and output. b. Administer calcium supplements. c. Assess lymph nodes for enlargement. d. Limit weight bearing and ambulation.

A high fluid intake and urine output helps prevent the complications of kidney stones caused by hypercalcemia and renal failure caused by deposition of Bence-Jones protein in the renal tubules. Weight bearing and ambulation are encouraged to help bone retain calcium. Lymph nodes are not enlarged with multiple myeloma. Calcium supplements will further increase the patient's calcium level and are not used

A 52-year-old male patient has received a bolus dose and an infusion of alteplase (Activase) for an ST-segment elevation myocardial infarction (STEMI). To determine the effectiveness of this medication, the nurse should assess the patient for the A presence of chest pain. B blood in the urine or stool. C tachycardia with hypotension. D decreased level of consciousness.

A presence of chest pain. Alteplase is a fibrinolytic that is administered to patients who have had an STEMI. If the medication is effective, the patient's chest pain will resolve because the medication dissolves the thrombus in the coronary artery and results in reperfusion of the myocardium. Bleeding is a major complication of fibrinolytic therapy. Signs of major bleeding include decreased level of consciousness, blood in the urine or stool, and increased heart rate with decreased blood pressure.

The nurse is reviewing the laboratory test results for a 68-year-old patient whose warfarin (Coumadin) therapy was terminated during the preoperative period. The nurse concludes that the patient is in the most stable condition for surgery after noting which INR (international normalized ratio) result? A. 1.0 B. 1.8 C. 2.7 D. 3.4

A. 1.0 The therapeutic range for INR is 2.0 to 3.0 for many clinical diagnoses. The larger the INR number, the greater the amount of anticoagulation. For this reason, the safest value before surgery is 1.0, meaning that the anticoagulation has been reversed.

Which person should the nurse identify as having the highest risk for abdominal aortic aneurysm? A. A 70-year-old male, with high cholesterol and hypertension B. A 40-year-old female with obesity and metabolic syndrome C. A 60-year-old male with renal insufficiency who is physically inactive D. A 65-year-old female with hyperhomocysteinemia and substance abuse

A. A 70-year-old male, with high cholesterol and hypertension The most common etiology of descending abdominal aortic aneurysm (AAA) is atherosclerosis. Male gender, age 65 years or older, and tobacco use are the major risk factors for AAAs of atherosclerotic origin. Other risk factors include the presence of coronary or peripheral artery disease, high blood pressure, and high cholesterol.

A 40-year-old man tells the nurse he has a diagnosis for the color and temperature changes of his limbs but can't remember the name of it. He says he must stop smoking and avoid trauma and exposure of his limbs to cold temperatures to get better. This description should allow the nurse to ask the patient if he has which diagnosis? A. Buerger's disease B. Venous thrombosis C. Acute arterial ischemia D. Raynaud's phenomenon

A. Buerger's disease Buerger's disease is a nonatherosclerotic, segmental, recurrent inflammatory disorder of small and medium-sized veins and arteries of upper and lower extremities leading to color and temperature changes of the limbs, intermittent claudication, rest pain, and ischemic ulcerations. It primarily occurs in men younger than 45 years old with a long history of tobacco and/or marijuana use. Buerger's disease treatment includes smoking cessation, trauma and cold temperature avoidance, and a walking program. Venous thrombosis is the formation of a thrombus in association with inflammation of the vein. Acute arterial ischemia is a sudden interruption in arterial blood flow to a tissue caused by embolism, thrombosis, or trauma. Raynaud's phenomenon is characterized by vasospasm-induced color changes of the fingers, toes, ears, and nose.

What medications should the nurse expect to include in the teaching plan to decrease the risk of cardiovascular events and death for PAD patients (select all that apply)? A. Ramipril (Altace) B. Cilostazol (Pletal) C. Simvastatin (Zocor) D. Clopidogrel (Plavix) E. Warfarin (Coumadin) F. Aspirin (acetylsalicylic acid)

A. Ramipril (Altace) C. Simvastatin (Zocor) F. Aspirin (acetylsalicylic acid) Angiotensin-converting enzyme inhibitors (e.g., ramipril [Altace]) are used to control hypertension. Statins (e.g., simvastatin [Zocor]) are used for lipid management. Aspirin is used as an antiplatelet agent. Cilostazol (Pletal) is used for intermittent claudication, but it does not reduce CVD morbidity and mortality risks. Clopidogrel may be used if the patient cannot tolerate aspirin. Anticoagulants (e.g., warfarin [Coumadin]) are not recommended to prevent CVD events in PAD patients.

Assessment of a patient's peripheral IV site reveals that phlebitis has developed over the past several hours. Which intervention should the nurse implement first? A. Remove the patient's IV catheter. B. Apply an ice pack to the affected area. C. Decrease the IV rate to 20 to 30 mL/hr. D. Administer prophylactic anticoagulants.

A. Remove the patient's IV catheter.

The nurse is admitting a 68-year-old preoperative patient with a suspected abdominal aortic aneurysm (AAA). The medication history reveals that the patient has been taking warfarin (Coumadin) on a daily basis. Based on this history and the patient's admission diagnosis, the nurse should prepare to administer which medication? A. Vitamin K B. Cobalamin C. Heparin sodium D. Protamine sulfate

A. Vitamin K Coumadin is a Vitamin K antagonist anticoagulant that could cause excessive bleeding during surgery if clotting times are not corrected before surgery. For this reason, vitamin K is given as the antidote for warfarin (Coumadin).

The nurse obtains a blood pressure of 176/83 mm Hg for a patient. What is the patient's mean arterial pressure (MAP)?

ANS: 114 mm Hg MAP = (SBP + 2 DBP)/3

A patient is to receive an infusion of 250 mL of platelets over 2 hours through tubing that is labeled: 1 mL equals 10 drops. How many drops per minute will the nurse infuse?

ANS: 21 To infuse 250 mL over 2 hours, the calculated drip rate is 20.8 drops/minute or 21 drops/minute

When analyzing an electrocardiographic (ECG) rhythm strip of a patient with a regular heart rhythm, the nurse counts 30 small blocks from one R wave to the next. The nurse calculates the patient's heart rate as ____.

ANS: 50 There are 1500 small blocks in a minute, and the nurse will divide 1500 by 30.

A patient's blood pressure in the postanesthesia care unit (PACU) has dropped from an admission blood pressure of 140/86 to 102/60 with a pulse change of 70 to 96. SpO2 is 92% on 3 L of oxygen. In which order should the nurse take these actions? (Put a comma and a space between each answer choice [A, B, C, D].) a. Increase the IV infusion rate. b. Assess the patient's dressing. c. Increase the oxygen flow rate. d. Check the patient's temperature

ANS: A, C, B, D The first nursing action should be to increase the IV infusion rate. Because the most common cause of hypotension is volume loss, the IV rate should be increased. The next action should be to increase the oxygen flow rate to maximize oxygenation of hypoperfused organs. Because hemorrhage is a common cause of postoperative volume loss, the nurse should check the dressing. Finally, the patient's temperature should be assessed to determine the effects of vasodilation caused by rewarming.

While ambulating in the room, a patient complains of feeling dizzy. In what order will the nurse accomplish the following activities? (Put a comma and a space between each answer choice [A, B, C, D].) a. Have the patient sit down in a chair. b. Give the patient something to drink. c. Take the patient's blood pressure (BP). d. Notify the patient's health care provider

ANS: A, C, B, D The first priority for the patient with syncope is to prevent a fall, so the patient should be assisted to a chair. Assessment of the BP will determine whether the dizziness is due to orthostatic hypotension, which occurs because of hypovolemia. Increasing the fluid intake will help prevent orthostatic dizziness. Because this is a common postoperative problem that is usually resolved through nursing measures such as increasing fluid intake and making position changes more slowly, there is no urgent need to notify the health care provider

When preparing to defibrillate a patient. In which order will the nurse perform the following steps? (Put a comma and a space between each answer choice [A, B, C, D, E].) a. Turn the defibrillator on. b. Deliver the electrical charge. c. Select the appropriate energy level. d. Place the paddles on the patient's chest. e. Check the location of other staff and call out "all clear."

ANS: A, C, D, E, B This order will result in rapid defibrillation without endangering hospital staff.

A patient with chronic pain who has been receiving morphine sulfate 20 mg IV over 24 hours is to be discharged home on oral sustained-release morphine (MS Contin), which will be administered twice a day. What dosage of MS Contin will be needed for each dose to obtain an equianalgesic dose for the patient? (Morphine sulfate 10 mg IV is equianalgesic to morphine sulfate 30 mg orally.)

ANS: MS Contin 30 mg/dose Morphine sulfate 20 mg IV over 24 hours will be equianalgesic to MS Contin 60 mg in 24 hours. Since the total dose needs to be divided into two doses, each dose should be 30 mg.

Which assessment finding in a patient admitted with acute decompensated heart failure (ADHF) requires the most immediate action by the nurse? a. Oxygen saturation of 88% b. Weight gain of 1 kg (2.2 lb) c. Heart rate of 106 beats/minute d. Urine output of 50 mL over 2 hours

ANS: A A decrease in oxygen saturation to less than 92% indicates hypoxemia. The nurse should administer supplemental oxygen immediately to the patient. An increase in apical pulse rate, 1-kg weight gain, and decreases in urine output also indicate worsening heart failure and require nursing actions, but the low oxygen saturation rate requires the most immediate nursing action

Which action will the nurse include in the plan of care for a patient who was admitted with syncopal episodes of unknown origin? a. Instruct the patient to call for assistance before getting out of bed. b. Explain the association between various dysrhythmias and syncope. c. Educate the patient about the need to avoid caffeine and other stimulants. d. Tell the patient about the benefits of implantable cardioverter-defibrillators.

ANS: A A patient with fainting episodes is at risk for falls. The nurse will plan to minimize the risk by having assistance whenever the patient up. The other actions may be needed if dysrhythmias are found to be the cause of the patient's syncope, but are not appropriate for syncope of unknown origin

When caring for a patient the second postoperative day after abdominal surgery for removal of a large pancreatic cyst, the nurse obtains an oral temperature of 100.8° F. Which action should the nurse take first? a. Have the patient use the incentive spirometer. b. Assess the surgical incision for redness and swelling. c. Administer the ordered PRN acetaminophen (Tylenol). d. Ask the health care provider to prescribe a different antibiotic.

ANS: A A temperature of 100.8° F in the first 48 hours is usually caused by atelectasis, and the nurse should have the patient cough and deep breathe. This problem may be resolved by nursing intervention, and therefore notifying the health care provider is not necessary. Acetaminophen will reduce the temperature, but it will not resolve the underlying respiratory congestion. Because a wound infection does not usually occur before the third postoperative day, a wound infection is not a likely source of the elevated temperature

Which action by a new registered nurse (RN) who is orienting to the progressive care unit indicates a good understanding of the treatment of cardiac dysrhythmias? a. Injects IV adenosine (Adenocard) over 2 seconds to a patient with supraventricular tachycardia b. Obtains the defibrillator and quickly brings it to the bedside of a patient whose monitor shows asystole c. Turns the synchronizer switch to the "on" position before defibrillating a patient with ventricular fibrillation d. Gives the prescribed dose of diltiazem (Cardizem) to a patient with new-onset type II second degree AV block

ANS: A Adenosine must be given over 1 to 2 seconds to be effective. The other actions indicate a need for more education about treatment of cardiac dysrhythmias. The RN should hold the diltiazem until talking to the health care provider. The treatment for asystole is immediate CPR. The synchronizer switch should be "off" when defibrillating

The nurse has just finished teaching a hypertensive patient about the newly prescribed ramipril (Altace). Which patient statement indicates that more teaching is needed? a. "A little swelling around my lips and face is okay." b. "The medication may not work as well if I take any aspirin." c. "The doctor may order a blood potassium level occasionally." d. "I will call the doctor if I notice that I have a frequent cough."

ANS: A Angioedema occurring with angiotensin-converting enzyme (ACE) inhibitor therapy is an indication that the ACE inhibitor should be discontinued. The patient should be taught that if any swelling of the face or oral mucosa occurs, the health care provider should be immediately notified because this could be life threatening. The other patient statements indicate that the patient has an accurate understanding of ACE inhibitor therapy

A patient with dilated cardiomyopathy has new onset atrial fibrillation that has been unresponsive to drug therapy for several days. The priority teaching needed for this patient would include information about a. anticoagulant therapy. b. permanent pacemakers. c. electrical cardioversion. d. IV adenosine (Adenocard).

ANS: A Atrial fibrillation therapy that has persisted for more than 48 hours requires anticoagulant treatment for 3 weeks before attempting cardioversion. This is done to prevent embolization of clots from the atria. Cardioversion may be done after several weeks of anticoagulation therapy. Adenosine is not used to treat atrial fibrillation. Pacemakers are routinely used for patients with bradydysrhythmias. Information does not indicate that the patient has a slow heart rate

After the nurse gives IV atropine to a patient with symptomatic type 1, second-degree atrioventricular (AV) block, which finding indicates that the medication has been effective? a. Increase in the patient's heart rate b. Increase in strength of peripheral pulses c. Decrease in premature atrial contractions d. Decrease in premature ventricular contractions

ANS: A Atropine will increase the heart rate and conduction through the AV node. Because the medication increases electrical conduction, not cardiac contractility, the quality of the peripheral pulses is not used to evaluate the drug effectiveness. The patient does not have premature atrial or ventricular contractions

A patient who uses a fentanyl (Duragesic) patch for chronic abdominal pain caused by ovarian cancer asks the nurse to administer the prescribed hydrocodone (Vicodin) tablets, but the patient is asleep when the nurse returns with the medication. Which action is best for the nurse to take? a. Wake the patient and administer the hydrocodone. b. Wait until the patient wakes up and reassess the pain. c. Suggest the use of nondrug therapies for pain relief instead of additional opioids. d. Consult with the health care provider about changing the fentanyl (Duragesic) dose.

ANS: A Because patients with chronic pain frequently use withdrawal and decreased activity as coping mechanisms for pain, sleep is not an indicator that the patient is pain free. The nurse should wake the patient and administer the hydrocodone

An appropriate nursing intervention for a patient with non-Hodgkin's lymphoma whose platelet count drops to 18,000/µL during chemotherapy is to a. check all stools for occult blood. b. encourage fluids to 3000 mL/day. c. provide oral hygiene every 2 hours. d. check the temperature every 4 hours.

ANS: A Because the patient is at risk for spontaneous bleeding, the nurse should check stools for occult blood. A low platelet count does not require an increased fluid intake. Oral hygiene is important, but it is not necessary to provide oral care every 2 hours. The low platelet count does not increase risk for infection, so frequent temperature monitoring is not indicated

The nurse is caring for a 78-year-old patient with aortic stenosis. Which assessment data obtained by the nurse would be most important to report to the health care provider? a. The patient complains of chest pressure when ambulating. b. A loud systolic murmur is heard along the right sternal border. c. A thrill is palpated at the second intercostal space, right sternal border. d. The point of maximum impulse (PMI) is at the left midclavicular line.

ANS: A Chest pressure (or pain) occurring with aortic stenosis is caused by cardiac ischemia, and reporting this information would be a priority. A systolic murmur and thrill are expected in a patient with aortic stenosis. A PMI at the left midclavicular line is normal

A patient with chronic back pain has learned to control the pain with the use of imagery and hypnosis. The patient's spouse asks the nurse how these techniques work. Which response by the nurse is best? a. "The strategies work by affecting the perception of pain." b. "These techniques block the pain pathways of the nerves." c. "Both strategies prevent transmission of painful stimuli to the brain." d. "The therapies slow the release of chemicals in the spinal cord that cause pain."

ANS: A Cognitive therapies affect the perception of pain by the brain rather than affecting efferent or afferent pathways or influencing the release of chemical transmitters in the dorsal horn

The nurse is caring for a 64-year-old patient admitted with mitral valve regurgitation. Which information obtained by the nurse when assessing the patient should be communicated to the health care provider immediately? a. The patient has bilateral crackles. b. The patient has bilateral, 4+ peripheral edema. c. The patient has a loud systolic murmur across the precordium. d. The patient has a palpable thrill felt over the left anterior chest.

ANS: A Crackles that are audible throughout the lungs indicate that the patient is experiencing severe left ventricular failure with pulmonary congestion and needs immediate interventions such as diuretics. A systolic murmur and palpable thrill would be expected in a patient with mitral regurgitation. Although 4+ peripheral edema indicates a need for a change in therapy, it does not need to be addressed urgently

A patient is admitted to the hospital with possible acute pericarditis. The nurse should plan to teach the patient about the purpose of a. echocardiography. b. daily blood cultures. c. cardiac catheterization. d. 24-hour Holter monitor.

ANS: A Echocardiograms are useful in detecting the presence of the pericardial effusions associated with pericarditis. Blood cultures are not indicated unless the patient has evidence of sepsis. Cardiac catheterization and 24-hour Holter monitor is not a diagnostic procedure for pericarditis

Which menu choice indicates that the patient understands the nurse's teaching about best dietary choices for iron-deficiency anemia? a. Omelet and whole wheat toast b. Cantaloupe and cottage cheese c. Strawberry and banana fruit plate d. Cornmeal muffin and orange juice

ANS: A Eggs and whole grain breads are high in iron. The other choices are appropriate for other nutritional deficiencies but are not the best choice for a patient with iron-deficiency anemia

The nurse examines the lymph nodes of a patient during a physical assessment. Which assessment finding would be of most concern to the nurse? a. A 2-cm nontender supraclavicular node b. A 1-cm mobile and nontender axillary node c. An inability to palpate any superficial lymph nodes d. Firm inguinal nodes in a patient with an infected foot

ANS: A Enlarged and nontender nodes are suggestive of malignancies such as lymphoma. Firm nodes are an expected finding in an area of infection. The superficial lymph nodes are usually not palpable in adults, but if they are palpable, they are normally 0.5 to 1 cm and nontender

To auscultate for S3 or S4 gallops in the mitral area, the nurse listens with the a. bell of the stethoscope with the patient in the left lateral position. b. diaphragm of the stethoscope with the patient in a supine position. c. bell of the stethoscope with the patient sitting and leaning forward. d. diaphragm of the stethoscope with the patient lying flat on the left side.

ANS: A Gallop rhythms generate low-pitched sounds and are most easily heard with the bell of the stethoscope. Sounds associated with the mitral valve are accentuated by turning the patient to the left side, which brings the heart closer to the chest wall. The diaphragm of the stethoscope is best to use for the higher-pitched sounds such as S1 and S2

Which information obtained by the nurse assessing a patient admitted with multiple myeloma is most important to report to the health care provider? a. Serum calcium level is 15 mg/dL. b. Patient reports no stool for 5 days. c. Urine sample has Bence-Jones protein. d. Patient is complaining of severe back pain.

ANS: A Hypercalcemia may lead to complications such as dysrhythmias or seizures, and should be addressed quickly. The other patient findings will also be discussed with the health care provider, but are not life threatening

A 20-year-old has a mandatory electrocardiogram (ECG) before participating on a college soccer team and is found to have sinus bradycardia, rate 52. Blood pressure (BP) is 114/54, and the student denies any health problems. What action by the nurse is most appropriate? a. Allow the student to participate on the soccer team. b. Refer the student to a cardiologist for further diagnostic testing. c. Tell the student to stop playing immediately if any dyspnea occurs. d. Obtain more detailed information about the student's family health history.

ANS: A In an aerobically trained individual, sinus bradycardia is normal. The student's normal BP and negative health history indicate that there is no need for a cardiology referral or for more detailed information about the family's health history. Dyspnea during an aerobic activity such as soccer is normal

Which patient statement to the nurse indicates a need for additional instruction about taking oral ferrous sulfate? a. "I will call my health care provider if my stools turn black." b. "I will take a stool softener if I feel constipated occasionally." c. "I should take the iron with orange juice about an hour before eating." d. "I should increase my fluid and fiber intake while I am taking iron tablets."

ANS: A It is normal for the stools to appear black when a patient is taking iron, and the patient should not call the doctor about this. The other patient statements are correct

A patient in the intensive care unit with acute decompensated heart failure (ADHF) complains of severe dyspnea and is anxious, tachypneic, and tachycardic. All of the following medications have been ordered for the patient. The nurse's priority action will be to a. give IV morphine sulfate 4 mg. b. give IV diazepam (Valium) 2.5 mg. c. increase nitroglycerin (Tridil) infusion by 5 mcg/min. d. increase dopamine (Intropin) infusion by 2 mcg/kg/min.

ANS: A Morphine improves alveolar gas exchange, improves cardiac output by reducing ventricular preload and afterload, decreases anxiety, and assists in reducing the subjective feeling of dyspnea. Diazepam may decrease patient anxiety, but it will not improve the cardiac output or gas exchange. Increasing the dopamine may improve cardiac output, but it will also increase the heart rate and myocardial oxygen consumption. Nitroglycerin will improve cardiac output and may be appropriate for this patient, but it will not directly reduce anxiety and will not act as quickly as morphine to decrease dyspnea

A patient who has just started taking sustained-release morphine sulfate (MS Contin) for chronic arthritic joint pain following a traumatic injury complains of nausea and abdominal fullness. Which action should the nurse take initially? a. Administer the ordered antiemetic medication. b. Tell the patient that the nausea will subside in about a week. c. Order the patient a clear liquid diet until the nausea decreases. d. Consult with the health care provider about using a different opioid.

ANS: A Nausea is frequently experienced with the initiation of opioid therapy, and antiemetics usually are prescribed to treat this expected side effect. There is no indication that a different opioid is needed, although if the nausea persists, the health care provider may order a change of opioid. Although tolerance develops and the nausea will subside in about a week, it is not appropriate to allow the patient to continue to be nauseated. A clear liquid diet may decrease the nausea, but the best choice would be to administer the antiemetic medication and allow the patient to eat

Which action should the nurse include in the plan of care when caring for a patient admitted with acute decompensated heart failure (ADHF) who is receiving nesiritide (Natrecor)? a. Monitor blood pressure frequently. b. Encourage patient to ambulate in room. c. Titrate nesiritide slowly before stopping. d. Teach patient about home use of the drug.

ANS: A Nesiritide is a potent arterial and venous dilator, and the major adverse effect is hypotension. Because the patient is likely to have orthostatic hypotension, the patient should not be encouraged to ambulate. Nesiritide does not require titration and is used for ADHF but not in a home setting

Propranolol (Inderal) is prescribed for a patient diagnosed with hypertension. The nurse should consult with the health care provider before giving this medication when the patient reveals a history of a. asthma. b. daily alcohol use. c. peptic ulcer disease. d. myocardial infarction (MI).

ANS: A Nonselective b-blockers block b1- and b2-adrenergic receptors and can cause bronchospasm, especially in patients with a history of asthma. b-Blockers will have no effect on the patient's peptic ulcer disease or alcohol use. b-Blocker therapy is recommended after MI

Which patient should the nurse assign as the roommate for a patient who has aplastic anemia? a. A patient with chronic heart failure b. A patient who has viral pneumonia c. A patient who has right leg cellulitis d. A patient with multiple abdominal drains

ANS: A Patients with aplastic anemia are at risk for infection because of the low white blood cell production associated with this type of anemia, so the nurse should avoid assigning a roommate with any possible infectious process

A 19-year-old woman with immune thrombocytopenic purpura (ITP) has an order for a platelet transfusion. Which information indicates that the nurse should consult with the health care provider before obtaining and administering platelets? a. The platelet count is 42,000/mL. b. Petechiae are present on the chest. c. Blood pressure (BP) is 94/56 mm Hg. d. Blood is oozing from the venipuncture site.

ANS: A Platelet transfusions are not usually indicated until the platelet count is below 10,000 to 20,000/mL unless the patient is actively bleeding. Therefore the nurse should clarify the order with the health care provider before giving the transfusion. The other data all indicate that bleeding caused by ITP may be occurring and that the platelet transfusion is appropriate

The nurse suspects cardiac tamponade in a patient who has acute pericarditis. To assess for the presence of pulsus paradoxus, the nurse should a. note when Korotkoff sounds are auscultated during both inspiration and expiration. b. subtract the diastolic blood pressure (DBP) from the systolic blood pressure (SBP). c. check the electrocardiogram (ECG) for variations in rate during the respiratory cycle. d. listen for a pericardial friction rub that persists when the patient is instructed to stop breathing.

ANS: A Pulsus paradoxus exists when there is a gap of greater than 10 mm Hg between when Korotkoff sounds can be heard during only expiration and when they can be heard throughout the respiratory cycle. The other methods described would not be useful in determining the presence of pulsus paradoxus

While caring for a patient with aortic stenosis, the nurse identifies a nursing diagnosis of acute pain related to decreased coronary blood flow. A priority nursing intervention for this patient would be to a. promote rest to decrease myocardial oxygen demand. b. teach the patient about the need for anticoagulant therapy. c. teach the patient to use sublingual nitroglycerin for chest pain. d. raise the head of the bed 60 degrees to decrease venous return.

ANS: A Rest is recommended to balance myocardial oxygen supply and demand and to decrease chest pain. The patient with aortic stenosis requires higher preload to maintain cardiac output, so nitroglycerin and measures to decrease venous return are contraindicated. Anticoagulation is not recommended unless the patient has atrial fibrillation

The nurse assesses a patient who has numerous petechiae on both arms. Which question should the nurse ask the patient? a. "Do you take salicylates?" b. "Are you taking any oral contraceptives?" c. "Have you been prescribed antiseizure drugs?" d. "How long have you taken antihypertensive drugs?"

ANS: A Salicylates interfere with platelet function and can lead to petechiae and ecchymoses. Antiseizure drugs may cause anemia, but not clotting disorders or bleeding. Oral contraceptives increase a person's clotting risk. Antihypertensives do not usually cause problems with decreased clotting

The charge nurse observes a new registered nurse (RN) doing discharge teaching for a patient with hypertension who has a new prescription for enalapril (Vasotec). The charge nurse will need to intervene if the new RN tells the patient to a. increase the dietary intake of high-potassium foods. b. make an appointment with the dietitian for teaching. c. check the blood pressure (BP) with a home BP monitor at least once a day. d. move slowly when moving from lying to sitting to standing.

ANS: A The ACE inhibitors cause retention of potassium by the kidney, so hyperkalemia is a possible adverse effect. The other teaching by the new RN is appropriate for a patient with newly diagnosed hypertension who has just started therapy with enalapril

To determine whether there is a delay in impulse conduction through the atria, the nurse will measure the duration of the patient's a. P wave. b. Q wave. c. P-R interval. d. QRS complex.

ANS: A The P wave represents the depolarization of the atria. The P-R interval represents depolarization of the atria, atrioventricular (AV) node, bundle of His, bundle branches, and the Purkinje fibers. The QRS represents ventricular depolarization. The Q wave is the first negative deflection following the P wave and should be narrow and short

The nurse hears a murmur between the S1 and S2 heart sounds at the patient's left fifth intercostal space and midclavicular line. How will the nurse record this information? a. Systolic murmur heard at mitral area b. Systolic murmur heard at Erb's point c. Diastolic murmur heard at aortic area d. Diastolic murmur heard at the point of maximal impulse

ANS: A The S1 signifies the onset of ventricular systole. S2 signifies the onset of diastole. A murmur occurring between these two sounds is a systolic murmur. The mitral area is the intersection of the left fifth intercostal space and the midclavicular line. The other responses describe murmurs heard at different landmarks on the chest and/or during the diastolic phase of the cardiac cycle

The nurse assesses a patient who had a total abdominal hysterectomy 2 days ago. Which information about the patient is most important to communicate to the health care provider? a. The right calf is swollen, warm, and painful. b. The patient's temperature is 100.3° F (37.9° C). c. The 24-hour oral intake is 600 mL greater than the total output. d. The patient complains of abdominal pain at level 6 (0 to 10 scale) when ambulating.

ANS: A The calf pain, swelling, and warmth suggest that the patient has a deep vein thrombosis, which will require the health care provider to order diagnostic tests and/or anticoagulants. Because the stress response causes fluid retention for the first 2 to 5 days postoperatively, the difference between intake and output is expected. A temperature elevation to 100.3° F on the second postoperative day suggests atelectasis, and the nurse should have the patient deep breathe and cough. Pain with ambulation is normal, and the nurse should administer the ordered analgesic before patient activities

The nurse is reviewing the laboratory test results for a patient who has recently been diagnosed with hypertension. Which result is most important to communicate to the health care provider? a. Serum creatinine of 2.8 mg/dL b. Serum potassium of 4.5 mEq/L c. Serum hemoglobin of 14.7 g/dL d. Blood glucose level of 96 mg/dL

ANS: A The elevated creatinine indicates renal damage caused by the hypertension. The other laboratory results are normal

A postoperative patient has not voided for 8 hours after return to the clinical unit. Which action should the nurse take first? a. Perform a bladder scan. b. Encourage increased oral fluid intake. c. Assist the patient to ambulate to the bathroom. d. Insert a straight catheter as indicated on the PRN order.

ANS: A The initial action should be to assess the bladder for distention. If the bladder is distended, providing the patient with privacy (by walking with them to the bathroom) will be helpful. Because of the risk for urinary tract infection, catheterization should only be done after other measures have been tried without success. There is no indication to notify the surgeon about this common postoperative problem unless all measures to empty the bladder are unsuccessful

The nurse caring for a patient with type A hemophilia being admitted to the hospital with severe pain and swelling in the right knee will a. immobilize the joint. b. apply heat to the knee. c. assist the patient with light weight bearing. d. perform passive range of motion to the knee.

ANS: A The initial action should be total rest of the knee to minimize bleeding. Ice packs are used to decrease bleeding. Range of motion (ROM) and weight-bearing exercise are contraindicated initially, but after the bleeding stops, ROM and physical therapy are started

Which nursing action should the nurse take first in order to assist a patient with newly diagnosed stage 1 hypertension in making needed dietary changes? a. Collect a detailed diet history. b. Provide a list of low-sodium foods. c. Help the patient make an appointment with a dietitian. d. Teach the patient about foods that are high in potassium.

ANS: A The initial nursing action should be assessment of the patient's baseline dietary intake through a thorough diet history. The other actions may be appropriate, but assessment of the patient's baseline should occur first

The standard policy on the cardiac unit states, "Notify the health care provider for mean arterial pressure (MAP) less than 70 mm Hg." The nurse will need to call the health care provider about the a. postoperative patient with a BP of 116/42. b. newly admitted patient with a BP of 150/87. c. patient with left ventricular failure who has a BP of 110/70. d. patient with a myocardial infarction who has a BP of 140/86.

ANS: A The mean arterial pressure (MAP) is calculated using the formula MAP = (systolic BP + 2 diastolic BP)/3. The MAP for the postoperative patient in answer 3 is 67. The MAP in the other three patients is higher than 70 mm Hg

The nurse working in the postanesthesia care unit (PACU) notes that a patient who has just been transported from the operating room is shivering and has a temperature of 96.5° F (35.8° C). Which action should the nurse take? a. Cover the patient with a warm blanket and put on socks. b. Notify the anesthesia care provider about the temperature. c. Avoid the use of opioid analgesics until the patient is warmer. d. Administer acetaminophen (Tylenol) 650 mg suppository rectally.

ANS: A The patient assessment indicates the need for active rewarming. There is no indication of a need for acetaminophen. Opioid analgesics may help reduce shivering. Because hypothermia is common in the immediate postoperative period, there is no need to notify the anesthesia care provider, unless the patient continues to be hypothermic after active rewarming

Which intervention by a new nurse who is caring for a patient who has just had an implantable cardioverter-defibrillator (ICD) inserted indicates a need for more education about care of patients with ICDs? a. The nurse assists the patient to do active range of motion exercises for all extremities. b. The nurse assists the patient to fill out the application for obtaining a Medic Alert ID. c. The nurse gives amiodarone (Cordarone) to the patient without first consulting with the health care provider. d. The nurse teaches the patient that sexual activity usually can be resumed once the surgical incision is healed.

ANS: A The patient should avoid moving the arm on the ICD insertion site until healing has occurred in order to prevent displacement of the ICD leads. The other actions by the new nurse are appropriate for this patient.

After receiving change-of-shift report on a heart failure unit, which patient should the nurse assess first? a. A patient who is cool and clammy, with new-onset confusion and restlessness b. A patient who has crackles bilaterally in the lung bases and is receiving oxygen. c. A patient who had dizziness after receiving the first dose of captopril (Capoten) d. A patient who is receiving IV nesiritide (Natrecor) and has a blood pressure of 100/62

ANS: A The patient who has "wet-cold" clinical manifestations of heart failure is perfusing inadequately and needs rapid assessment and changes in management. The other patients also should be assessed as quickly as possible but do not have indications of severe decreases in tissue perfusion.

The nurse on the intermediate care unit received change-of-shift report on four patients with hypertension. Which patient should the nurse assess first? a. 43-year-old with a (blood pressure (BP) of 160/92 who is complaining of chest pain b. 52-year-old with a BP of 212/90 who has intermittent claudication c. 50-year-old with a BP of 190/104 who has a creatinine of 1.7 mg/dL d. 48-year-old with a BP of 172/98 whose urine shows microalbuminuria

ANS: A The patient with chest pain may be experiencing acute myocardial infarction, and rapid assessment and intervention are needed. The symptoms of the other patients also show target organ damage but are not indicative of acute processes

Two days after an acute myocardial infarction (MI), a patient complains of stabbing chest pain that increases with a deep breath. Which action will the nurse take first? a. Auscultate the heart sounds. b. Check the patient's temperature. c. Notify the patient's health care provider. d. Give the PRN acetaminophen (Tylenol).

ANS: A The patient's clinical manifestations and history are consistent with pericarditis, and the first action by the nurse should be to listen for a pericardial friction rub. Checking the temperature and notifying the health care provider are also appropriate actions but would not be done before listening for a rub. It is not stated for what symptom (e.g., headache) or finding (e.g., increased temperature) the PRN acetaminophen (Tylenol) is ordered.

After receiving change-of-shift report about these postoperative patients, which patient should the nurse assess first? a. Obese patient who had abdominal surgery 3 days ago and whose wound edges are separating b. Patient who has 30 mL of sanguineous drainage in the wound drain 10 hours after hip replacement surgery c. Patient who has bibasilar crackles and a temperature of 100°F (37.8°C) on the first postoperative day after chest surgery d. Patient who continues to have incisional pain 15 minutes after hydrocodone and acetaminophen (Vicodin) administration

ANS: A The patient's history and assessment suggests possible wound dehiscence, which should be reported immediately to the surgeon. Although the information about the other patients indicates a need for ongoing assessment and/or possible intervention, the data do not suggest any acute complications. Small amounts of red drainage are common in the first postoperative hours. Bibasilar crackles and a slightly elevated temperature are common after surgery, although the nurse will need to have the patient cough and deep breathe. Oral medications typically take more than 15 minutes for effective pain relief

The nurse is caring for a 1-day postoperative patient who is receiving morphine through patient-controlled analgesia (PCA). What action by the nurse is a priority? a. Check the respiratory rate. b. Assess for nausea after eating. c. Inspect the abdomen and auscultate bowel sounds. d. Evaluate the sacral and heel areas for signs of redness.

ANS: A The patient's respiratory rate is the highest priority of care while using PCA medication because of the possible respiratory depression. The other information may also require intervention but is not as urgent to report as the respiratory rate

A patient's cardiac monitor shows a pattern of undulations of varying contours and amplitude with no measurable ECG pattern. The patient is unconscious and pulseless. Which action should the nurse take first? a. Perform immediate defibrillation. b. Give epinephrine (Adrenalin) IV. c. Prepare for endotracheal intubation. d. Give ventilations with a bag-valve-mask device.

ANS: A The patient's rhythm and assessment indicate ventricular fibrillation and cardiac arrest; the initial action should be to defibrillate. If a defibrillator is not immediately available or is unsuccessful in converting the patient to a better rhythm, the other actions may be appropriate

During a visit to a 78-year-old with chronic heart failure, the home care nurse finds that the patient has ankle edema, a 2-kg weight gain over the past 2 days, and complains of "feeling too tired to get out of bed." Based on these data, the best nursing diagnosis for the patient is a. activity intolerance related to fatigue. b. disturbed body image related to weight gain. c. impaired skin integrity related to ankle edema. d. impaired gas exchange related to dyspnea on exertion.

ANS: A The patient's statement supports the diagnosis of activity intolerance. There are no data to support the other diagnoses, although the nurse will need to assess for other patient problems

A patient who had knee surgery received intramuscular ketorolac (Toradol) 30 minutes ago and continues to complain of pain at a level of 7 (0 to 10 scale). Which action is best for the nurse to take at this time? a. Administer the prescribed PRN IV morphine sulfate. b. Notify the health care provider about the ongoing knee pain. c. Reassure the patient that postoperative pain is expected after knee surgery. d. Teach the patient that the effects of ketorolac typically last about 6 to 8 hours.

ANS: A The priority at this time is pain relief. Concomitant use of opioids and nonsteroidal antiinflammatory drugs (NSAIDs) improves pain control in postoperative patients. Patient teaching and reassurance are appropriate, but should be done after the patient's pain is relieved. If the patient continues to have pain after the morphine is administered, the health care provider should be notified

The complete blood count (CBC) indicates that a patient is thrombocytopenic. Which action should the nurse include in the plan of care? a. Avoid intramuscular injections. b. Encourage increased oral fluids. c. Check temperature every 4 hours. d. Increase intake of iron-rich foods.

ANS: A Thrombocytopenia is a decreased number of platelets, which places the patient at high risk for bleeding. Neutropenic patients are at high risk for infection and sepsis and should be monitored frequently for signs of infection. Encouraging fluid intake and iron-rich food intake is not indicated in a patient with thrombocytopenia

A nurse assesses a postoperative patient 2 days after chest surgery. What findings indicate that the patient requires better pain management (select all that apply)? a. Confusion b. Hypoglycemia c. Poor cough effort d. Shallow breathing e. Elevated temperature

ANS: A, C, D, E Inadequate pain control can decrease tidal volume and cough effort, leading to complications such as pneumonia with increases in temperature. Poor pain control may lead to confusion through a variety of mechanism, including hypoventilation and poor sleep quality. Stressors such as pain cause increased release of corticosteroids that can result in hyperglycemia.

Based on the Joint Commission Core Measures for patients with heart failure, which topics should the nurse include in the discharge teaching plan for a patient who has been hospitalized with chronic heart failure (select all that apply)? a. How to take and record daily weight b. Importance of limiting aerobic exercise c. Date and time of follow-up appointment d. Symptoms indicating worsening heart failure e. Actions and side effects of prescribed medications

ANS: A, C, D, E The Joint Commission Core Measures state that patients should be taught about prescribed medications, follow-up appointments, weight monitoring, and actions to take for worsening symptoms. Patients with heart failure are encouraged to begin or continue aerobic exercises such as walking, while self-monitoring to avoid excessive fatigue

A patient with pancytopenia of unknown origin is scheduled for the following diagnostic tests. The nurse will provide a consent form to sign for which test? a. ABO blood typing b. Bone marrow biopsy c. Abdominal ultrasound d. Complete blood count (CBC)

ANS: B A bone marrow biopsy is a minor surgical procedure that requires the patient or guardian to sign a surgical consent form. The other procedures do not require a signed consent by the patient or guardian

When auscultating over the patient's abdominal aorta, the nurse hears a humming sound. The nurse documents this finding as a a. thrill. b. bruit. c. murmur. d. normal finding.

ANS: B A bruit is the sound created by turbulent blood flow in an artery. Thrills are palpable vibrations felt when there is turbulent blood flow through the heart or in a blood vessel. A murmur is the sound caused by turbulent blood flow through the heart. Auscultating a bruit in an artery is not normal and indicates pathology

Which patient requires the most rapid assessment and care by the emergency department nurse? a. The patient with hemochromatosis who reports abdominal pain b. The patient with neutropenia who has a temperature of 101.8° F c. The patient with sickle cell anemia who has had nausea and diarrhea for 24 hours d. The patient with thrombocytopenia who has oozing after having a tooth extracted

ANS: B A neutropenic patient with a fever is assumed to have an infection and is at risk for rapidly developing sepsis. Rapid assessment, cultures, and initiation of antibiotic therapy are needed. The other patients also require rapid assessment and care but not as urgently as the neutropenic patient

A patient is scheduled for a cardiac catheterization with coronary angiography. Before the test, the nurse informs the patient that a. it will be important to lie completely still during the procedure. b. a flushed feeling may be noted when the contrast dye is injected. c. monitored anesthesia care will be provided during the procedure. d. arterial pressure monitoring will be required for 24 hours after the test.

ANS: B A sensation of warmth or flushing is common when the contrast material is injected, which can be anxiety-producing unless it has been discussed with the patient. The patient may receive a sedative drug before the procedure, but monitored anesthesia care is not used. Arterial pressure monitoring is not routinely used after the procedure to monitor blood pressure. The patient is not immobile during cardiac catheterization and may be asked to cough or take deep breaths

On admission of a patient to the postanesthesia care unit (PACU), the blood pressure (BP) is 122/72. Thirty minutes after admission, the BP falls to 114/62, with a pulse of 74 and warm, dry skin. Which action by the nurse is most appropriate? a. Increase the IV fluid rate. b. Continue to take vital signs every 15 minutes. c. Administer oxygen therapy at 100% per mask. d. Notify the anesthesia care provider (ACP) immediately.

ANS: B A slight drop in postoperative BP with a normal pulse and warm, dry skin indicates normal response to the residual effects of anesthesia and requires only ongoing monitoring. Hypotension with tachycardia and/or cool, clammy skin would suggest hypovolemic or hemorrhagic shock and the need for notification of the ACP, increased fluids, and high-concentration oxygen administration

The nurse is reviewing laboratory results and notes an aPTT level of 28 seconds. The nurse should notify the health care provider in anticipation of adjusting which medication? a. Aspirin b. Heparin c. Warfarin d. Erythropoietin

ANS: B Activated partial thromboplastin time (aPTT) assesses intrinsic coagulation by measuring factors I, II, V, VIII, IX, X, XI, XII. aPTT is increased (prolonged) in heparin administration. aPTT is used to monitor whether heparin is at a therapeutic level (needs to be greater than the normal range of 25 to 35 sec). Prothrombin time (PT) and international normalized ratio (INR) are most commonly used to test for therapeutic levels of warfarin (Coumadin). Aspirin affects platelet function. Erythropoietin is used to stimulate red blood cell production

A nurse assists a patient on the first postoperative day to ambulate, cough, deep breathe, and turn. Which action by the nurse is most helpful? a. Teach the patient to fully exhale into the incentive spirometer. b. Administer ordered analgesic medications before these activities. c. Ask the patient to state two possible complications of immobility. d. Encourage the patient to state the purpose of splinting the incision.

ANS: B An important nursing action to encourage these postoperative activities is administration of adequate analgesia to allow the patient to accomplish the activities with minimal pain. Even with motivation provided by proper teaching, positive reinforcement, and concern about complications, patients will have difficulty if there is a great deal of pain involved with these activities. When using an incentive spirometer, the patient should be taught to inhale deeply, rather than exhale into the spirometer to promote lung expansion and prevent atelectasis

After receiving change-of-shift report for several patients with neutropenia, which patient should the nurse assess first? a. 56-year-old with frequent explosive diarrhea b. 33-year-old with a fever of 100.8° F (38.2° C) c. 66-year-old who has white pharyngeal lesions d. 23-year old who is complaining of severe fatigue

ANS: B Any fever in a neutropenic patient indicates infection and can quickly lead to sepsis and septic shock. Rapid assessment and (if prescribed) initiation of antibiotic therapy within 1 hour are needed. The other patients also need to be assessed but do not exhibit symptoms of potentially life-threatening problems

A patient with hypertension who has just started taking atenolol (Tenormin) returns to the health clinic after 2 weeks for a follow-up visit. The blood pressure (BP) is unchanged from the previous visit. Which action should the nurse take first? a. Inform the patient about the reasons for a possible change in drug dosage. b. Question the patient about whether the medication is actually being taken. c. Inform the patient that multiple drugs are often needed to treat hypertension. d. Question the patient regarding any lifestyle changes made to help control BP.

ANS: B Because noncompliance with antihypertensive therapy is common, the nurse's initial action should be to determine whether the patient is taking the atenolol as prescribed. The other actions also may be implemented, but these would be done after assessing patient compliance with the prescribed therapy

Which question asked by the nurse will give the most information about the patient's metastatic bone cancer pain? a. "How long have you had this pain?" b. "How would you describe your pain?" c. "How much medication do you take for the pain?" d. "How many times a day do you take medication for the pain?"

ANS: B Because pain is a multidimensional experience, asking a question that addresses the patient's experience with the pain will elicit more information than the more specific information asked in the other three responses. All of these questions are appropriate, but the response beginning "How would you describe your pain?" is the best initial question

The nurse reviews the laboratory results for a patient on the first postoperative day after a hiatal hernia repair. Which finding would indicate to the nurse that the patient is at increased risk for poor wound healing? a. Potassium 3.5 mEq/L b. Albumin level 2.2 g/dL c. Hemoglobin 11.2 g/dL d. White blood cells 11,900/µL

ANS: B Because proteins are needed for an appropriate inflammatory response and wound healing, the low serum albumin level (normal level 3.5 to 5.0 g/dL) indicates a risk for poor wound healing. The potassium level is normal. Because a small amount of blood loss is expected with surgery, the hemoglobin level is not indicative of an increased risk for wound healing. WBC count is expected to increase after surgery as a part of the normal inflammatory response

Which collaborative problem will the nurse include in a care plan for a patient admitted to the hospital with idiopathic aplastic anemia? a. Potential complication: seizures b. Potential complication: infection c. Potential complication: neurogenic shock d. Potential complication: pulmonary edema

ANS: B Because the patient with aplastic anemia has pancytopenia, the patient is at risk for infection and bleeding. There is no increased risk for seizures, neurogenic shock, or pulmonary edema

A patient who has begun to awaken after 30 minutes in the postanesthesia care unit (PACU) is restless and shouting at the nurse. The patient's oxygen saturation is 96%, and recent laboratory results are all normal. Which action by the nurse is most appropriate? a. Increase the IV fluid rate. b. Assess for bladder distention. c. Notify the anesthesia care provider (ACP). d. Demonstrate the use of the nurse call bell button.

ANS: B Because the patient's assessment indicates physiologic stability, the most likely cause of the patient's agitation is emergence delirium, which will resolve as the patient wakes up more fully. The nurse should look for a cause such as bladder distention. Although hypoxemia is the most common cause, the patient's oxygen saturation is 96%. Emergence delirium is common in patients recovering from anesthesia, so there is no need to notify the ACP. Orientation of the patient to bed controls is needed, but is not likely to be effective until the effects of anesthesia have resolved more completely

A routine complete blood count indicates that an active 80-year-old man may have myelodysplastic syndrome. The nurse will plan to teach the patient about a. blood transfusion b. bone marrow biopsy. c. filgrastim (Neupogen) administration. d. erythropoietin (Epogen) administration.

ANS: B Bone marrow biopsy is needed to make the diagnosis and determine the specific type of myelodysplastic syndrome. The other treatments may be necessary if there is progression of the myelodysplastic syndrome, but the initial action for this asymptomatic patient will be a bone marrow biopsy.

The nurse identifies the nursing diagnosis of decreased cardiac output related to valvular insufficiency for the patient with infective endocarditis (IE) based on which assessment finding(s)? a. Fever, chills, and diaphoresis b. Urine output less than 30 mL/hr c. Petechiae on the inside of the mouth and conjunctiva d. Increase in heart rate of 15 beats/minute with walking

ANS: B Decreased renal perfusion caused by inadequate cardiac output will lead to decreased urine output. Petechiae, fever, chills, and diaphoresis are symptoms of IE, but are not caused by decreased cardiac output. An increase in pulse rate of 15 beats/minute is normal with exercise

Which information obtained by the nurse caring for a patient with thrombocytopenia should be immediately communicated to the health care provider? a. The platelet count is 52,000/µL. b. The patient is difficult to arouse. c. There are purpura on the oral mucosa. d. There are large bruises on the patient's back.

ANS: B Difficulty in arousing the patient may indicate a cerebral hemorrhage, which is life threatening and requires immediate action. The other information should be documented and reported but would not be unusual in a patient with thrombocytopenia

A patient who is just waking up after having hip replacement surgery is agitated and confused. Which action should the nurse take first? a. Administer the ordered opioid. b. Check the oxygen (O2) saturation. c. Take the blood pressure and pulse. d. Apply wrist restraints to secure IV lines.

ANS: B Emergence delirium may be caused by a variety of factors. However, the nurse should first assess for hypoxemia. The other actions also may be appropriate, but are not the best initial action

Which assessment finding for a patient who is receiving IV furosemide (Lasix) to treat stage 2 hypertension is most important to report to the health care provider? a. Blood glucose level of 175 mg/dL b. Blood potassium level of 3.0 mEq/L c. Most recent blood pressure (BP) reading of 168/94 mm Hg d. Orthostatic systolic BP decrease of 12 mm Hg

ANS: B Hypokalemia is a frequent adverse effect of the loop diuretics and can cause life-threatening dysrhythmias. The health care provider should be notified of the potassium level immediately and administration of potassium supplements initiated. The elevated blood glucose and BP also indicate a need for collaborative interventions but will not require action as urgently as the hypokalemia. An orthostatic drop of 12 mm Hg is common and will require intervention only if the patient is symptomatic

After receiving report on the following patients, which patient should the nurse assess first? a. Patient with rheumatic fever who has sharp chest pain with a deep breath b. Patient with acute aortic regurgitation whose blood pressure is 86/54 mm Hg c. Patient with infective endocarditis who has a murmur and splinter hemorrhages d. Patient with dilated cardiomyopathy who has bilateral crackles at the lung bases

ANS: B Hypotension in patients with acute aortic regurgitation may indicate cardiogenic shock. The nurse should immediately assess this patient for other findings such as dyspnea or chest pain. The findings in the other patients are typical of their diagnoses and do not indicate a need for urgent assessment and intervention

Which intervention will be included in the nursing care plan for a patient with immune thrombocytopenic purpura (ITP)? a. Assign the patient to a private room. b. Avoid intramuscular (IM) injections. c. Use rinses rather than a soft toothbrush for oral care. d. Restrict activity to passive and active range of motion.

ANS: B IM or subcutaneous injections should be avoided because of the risk for bleeding. A soft toothbrush can be used for oral care. There is no need to restrict activity or place the patient in a private room

A 19-year-old student comes to the student health center at the end of the semester complaining that, "My heart is skipping beats." An electrocardiogram (ECG) shows occasional premature ventricular contractions (PVCs). What action should the nurse take next? a. Start supplemental O2 at 2 to 3 L/min via nasal cannula. b. Ask the patient about current stress level and caffeine use. c. Ask the patient about any history of coronary artery disease. d. Have the patient taken to the hospital emergency department (ED).

ANS: B In a patient with a normal heart, occasional PVCs are a benign finding. The timing of the PVCs suggests stress or caffeine as possible etiologic factors. It is unlikely that the patient has coronary artery disease, and this should not be the first question the nurse asks. The patient is hemodynamically stable, so there is no indication that the patient needs to be seen in the ED or that oxygen needs to be administered

A 53-year-old patient with Stage D heart failure and type 2 diabetes asks the nurse whether heart transplant is a possible therapy. Which response by the nurse is most appropriate? a. "Because you have diabetes, you would not be a candidate for a heart transplant." b. "The choice of a patient for a heart transplant depends on many different factors." c. "Your heart failure has not reached the stage in which heart transplants are needed." d. "People who have heart transplants are at risk for multiple complications after surgery."

ANS: B Indications for a heart transplant include end-stage heart failure (Stage D), but other factors such as coping skills, family support, and patient motivation to follow the rigorous posttransplant regimen are also considered. Diabetic patients who have well-controlled blood glucose levels may be candidates for heart transplant. Although heart transplants can be associated with many complications, this response does not address the patient's question

The nurse notes scleral jaundice in a patient being admitted with hemolytic anemia. The nurse will plan to check the laboratory results for the a. Schilling test. b. bilirubin level. c. stool occult blood test. d. gastric analysis testing.

ANS: B Jaundice is caused by the elevation of bilirubin level associated with red blood cell (RBC) hemolysis. The other tests would not be helpful in monitoring or treating a hemolytic anemia

A patient has ST segment changes that support an acute inferior wall myocardial infarction. Which lead would be best for monitoring the patient? a. I b. II c. V2 d. V6

ANS: B Leads II, III, and AVF reflect the inferior area of the heart and the ST segment changes. Lead II will best capture any electrocardiographic (ECG) changes that indicate further damage to the myocardium. The other leads do not reflect the inferior part of the myocardial wall and will not provide data about further ischemic changes in that area

Which problem reported by a patient with hemophilia is most important for the nurse to communicate to the physician? a. Leg bruises b. Tarry stools c. Skin abrasions d. Bleeding gums

ANS: B Melena is a sign of gastrointestinal bleeding and requires collaborative actions such as checking hemoglobin and hematocrit and administration of coagulation factors. The other problems indicate a need for patient teaching about how to avoid injury, but are not indicators of possible serious blood loss

A patient who is receiving methotrexate for severe rheumatoid arthritis develops a megaloblastic anemia. The nurse will anticipate teaching the patient about increasing oral intake of a. iron. b. folic acid. c. cobalamin (vitamin B12). d. ascorbic acid (vitamin C).

ANS: B Methotrexate use can lead to folic acid deficiency. Supplementation with oral folic acid supplements is the usual treatment. The other nutrients would not correct folic acid deficiency, although they would be used to treat other types of anemia

When assessing a newly admitted patient, the nurse notes a murmur along the left sternal border. To document more information about the murmur, which action will the nurse take next? a. Find the point of maximal impulse. b. Determine the timing of the murmur. c. Compare the apical and radial pulse rates. d. Palpate the quality of the peripheral pulses.

ANS: B Murmurs are caused by turbulent blood flow, such as occurs when blood flows through a damaged valve. Relevant information includes the position in which the murmur is heard best (e.g., sitting and leaning forward), the timing of the murmur in relation to the cardiac cycle (e.g., systole, diastole), and where on the thorax the murmur is heard best. The other information is also important in the cardiac assessment but will not provide information that is relevant to the murmur

The nurse is assessing a patient with myocarditis before administering the scheduled dose of digoxin (Lanoxin). Which finding is most important for the nurse to communicate to the health care provider? a. Leukocytosis b. Irregular pulse c. Generalized myalgia d. Complaint of fatigue

ANS: B Myocarditis predisposes the heart to digoxin-associated dysrhythmias and toxicity. The other findings are common symptoms of myocarditis and there is no urgent need to report these.

During the assessment of a 25-year-old patient with infective endocarditis (IE), the nurse would expect to find a. substernal chest pressure. b. a new regurgitant murmur. c. a pruritic rash on the chest. d. involuntary muscle movement.

ANS: B New regurgitant murmurs occur in IE because vegetations on the valves prevent valve closure. Substernal chest discomfort, rashes, and involuntary muscle movement are clinical manifestations of other cardiac disorders such as angina and rheumatic fever

An appropriate nursing intervention for a hospitalized patient with severe hemolytic anemia is to a. provide a diet high in vitamin K. b. alternate periods of rest and activity. c. teach the patient how to avoid injury. d. place the patient on protective isolation.

ANS: B Nursing care for patients with anemia should alternate periods of rest and activity to encourage activity without causing undue fatigue. There is no indication that the patient has a bleeding disorder, so a diet high in vitamin K or teaching about how to avoid injury is not needed. Protective isolation might be used for a patient with aplastic anemia, but it is not indicated for hemolytic anemia

Which nursing action could the nurse delegate to unlicensed assistive personnel (UAP) when caring for a patient who is using a fentanyl (Duragesic) patch and a heating pad for treatment of chronic back pain? a. Check the skin under the heating pad. b. Take the respiratory rate every 2 hours. c. Monitor sedation using the sedation assessment scale. d. Ask the patient about whether pain control is effective.

ANS: B Obtaining the respiratory rate is included in UAP education and scope of practice. Assessment for sedation, pain control, and skin integrity requires more education and scope of practice

The following medications are prescribed by the health care provider for a middle-aged patient who uses long-acting morphine (MS Contin) for chronic back pain, but still has ongoing pain. Which medication should the nurse question? a. Morphine (Roxanol) b. Pentazocine (Talwin) c. Celecoxib (Celebrex) d. Dexamethasone (Decadron)

ANS: B Opioid agonist-antagonists can precipitate withdrawal if used in a patient who is physically dependent on mu agonist drugs such as morphine. The other medications are appropriate for the patient

A patient with second-degree burns has been receiving hydromorphone through patient-controlled analgesia (PCA) for a week. The patient wakes up frequently during the night complaining of pain. What action by the nurse is most appropriate? a. Administer a dose of morphine every 1 to 2 hours from the PCA machine while the patient is sleeping. b. Consult with the health care provider about using a different treatment protocol to control the patient's pain. c. Request that the health care provider order a bolus dose of morphine to be given when the patient awakens with pain. d. Teach the patient to push the button every 10 minutes for an hour before going to sleep, even if the pain is minimal.

ANS: B PCAs are best for controlling acute pain. This patient's history indicates chronic pain and a need for a pain management plan that will provide adequate analgesia while the patient is sleeping. Administering a dose of morphine when the patient already has severe pain will not address the problem. Teaching the patient to administer unneeded medication before going to sleep can result in oversedation and respiratory depression. It is illegal for the nurse to administer the morphine for a patient through PCA.

It is important for the nurse providing care for a patient with sickle cell crisis to a. limit the patient's intake of oral and IV fluids. b. evaluate the effectiveness of opioid analgesics. c. encourage the patient to ambulate as much as tolerated. d. teach the patient about high-protein, high-calorie foods.

ANS: B Pain is the most common clinical manifestation of a crisis and usually requires large doses of continuous opioids for control. Fluid intake should be increased to reduce blood viscosity and improve perfusion. Rest is usually ordered to decrease metabolic requirements. Patients are instructed about the need for dietary folic acid, but high-protein, high-calorie diets are not emphasized

To assess the patient with pericarditis for evidence of a pericardial friction rub, the nurse should a. listen for a rumbling, low-pitched, systolic murmur over the left anterior chest. b. auscultate by placing the diaphragm of the stethoscope on the lower left sternal border. c. ask the patient to cough during auscultation to distinguish the sound from a pleural friction rub. d. feel the precordial area with the palm of the hand to detect vibrations with cardiac contraction.

ANS: B Pericardial friction rubs are heard best with the diaphragm at the lower left sternal border. The nurse should ask the patient to hold his or her breath during auscultation to distinguish the sounds from a pleural friction rub. Friction rubs are not typically low pitched or rumbling and are not confined to systole. Rubs are not assessed by palpation

The nurse is admitting a patient with possible rheumatic fever. Which question on the admission health history will be most pertinent to ask? a. "Do you use any illegal IV drugs?" b. "Have you had a recent sore throat?" c. "Have you injured your chest in the last few weeks?" d. "Do you have a family history of congenital heart disease?"

ANS: B Rheumatic fever occurs as a result of an abnormal immune response to a streptococcal infection. Although illicit IV drug use should be discussed with the patient before discharge, it is not a risk factor for rheumatic fever, and would not be as pertinent when admitting the patient. Family history is not a risk factor for rheumatic fever. Chest injury would cause musculoskeletal chest pain rather than rheumatic fever.

Which assessment finding in a patient who is admitted with infective endocarditis (IE) is most important to communicate to the health care provider? a. Generalized muscle aching b. Sudden onset right flank pain c. Janeway's lesions on the palms d. Temperature 100.7° F (38.1° C)

ANS: B Sudden onset of flank pain indicates possible embolization to the kidney and may require diagnostic testing such as a renal arteriogram and interventions to improve renal perfusion. The other findings are typically found in IE, but do not require any new interventions

A patient with a history of hypertension treated with a diuretic and an angiotensin-converting enzyme (ACE) inhibitor arrives in the emergency department complaining of a severe headache and nausea and has a blood pressure (BP) of 238/118 mm Hg. Which question should the nurse ask first? a. "Did you take any acetaminophen (Tylenol) today?" b. "Have you been consistently taking your medications?" c. "Have there been any recent stressful events in your life?" d. "Have you recently taken any antihistamine medications?"

ANS: B Sudden withdrawal of antihypertensive medications can cause rebound hypertension and hypertensive crisis. Although many over-the-counter medications can cause hypertension, antihistamines and acetaminophen do not increase BP. Stressful events will increase BP but not usually to the level seen in this patient

A patient with a history of a transfusion-related acute lung injury (TRALI) is to receive a transfusion of packed red blood cells (PRBCs). Which action by the nurse will decrease the risk for TRALI for this patient? a. Infuse the PRBCs slowly over 4 hours. b. Transfuse only leukocyte-reduced PRBCs. c. Administer the scheduled diuretic before the transfusion. d. Give the PRN dose of antihistamine before the transfusion.

ANS: B TRALI is caused by a reaction between the donor and the patient leukocytes that causes pulmonary inflammation and capillary leaking. The other actions may help prevent respiratory problems caused by circulatory overload or by allergic reactions, but they will not prevent TRALI

A patient who is using a fentanyl (Duragesic) patch and immediate-release morphine for chronic cancer pain develops new-onset confusion, dizziness, and a decrease in respiratory rate. Which action should the nurse take first? a. Obtain vital signs. b. Remove the fentanyl patch. c. Notify the health care provider. d. Administer the prescribed PRN naloxone (Narcan).

ANS: B The assessment data indicate a possible overdose of opioid. The first action should be to remove the patch. Naloxone administration in a patient who has been chronically using opioids can precipitate withdrawal and would not be the first action. Notification of the health care provider and continued monitoring are also needed, but the patient's data indicate that more rapid action is needed. The respiratory rate alone is an indicator for immediate action before obtaining blood pressure, pulse, and temperature

A 28-year-old man with von Willebrand disease is admitted to the hospital for minor knee surgery. The nurse will review the coagulation survey to check the a. platelet count. b. bleeding time. c. thrombin time. d. prothrombin time.

ANS: B The bleeding time is affected by von Willebrand disease. Platelet count, prothrombin time, and thrombin time are normal in von Willebrand disease

A registered nurse (RN) is observing a student nurse who is doing a physical assessment on a patient. The RN will need to intervene immediately if the student nurse a. presses on the skin over the tibia for 10 seconds to check for edema. b. palpates both carotid arteries simultaneously to compare pulse quality. c. documents a murmur heard along the right sternal border as a pulmonic murmur. d. places the patient in the left lateral position to check for the point of maximal impulse.

ANS: B The carotid pulses should never be palpated at the same time to avoid vagal stimulation, dysrhythmias, and decreased cerebral blood flow. The other assessment techniques also need to be corrected. However, they are not dangerous to the patient

When admitting a patient for a cardiac catheterization and coronary angiogram, which information about the patient is most important for the nurse to communicate to the health care provider? a. The patient's pedal pulses are +1. b. The patient is allergic to shellfish. c. The patient had a heart attack a year ago. d. The patient has not eaten anything today.

ANS: B The contrast dye used for the procedure is iodine based, so patients who have shellfish allergies will require treatment with medications such as corticosteroids and antihistamines before the angiogram. The other information is also communicated to the health care provider but will not require a change in the usual precardiac catheterization orders or medications

Which action will the admitting nurse include in the care plan for a 30-year old woman who is neutropenic? a. Avoid any injections. b. Check temperature every 4 hours. c. Omit fruits or vegetables from the diet. d. Place a "No Visitors" sign on the door.

ANS: B The earliest sign of infection in a neutropenic patient is an elevation in temperature. Although unpeeled fresh fruits and vegetables should be avoided, fruits and vegetables that are peeled or cooked are acceptable. Injections may be required for administration of medications such as filgrastim (Neupogen). The number of visitors may be limited and visitors with communicable diseases should be avoided, but a "no visitors" policy is not needed

Which blood pressure (BP) finding by the nurse indicates that no changes in therapy are needed for a patient with stage 1 hypertension who has a history of diabetes mellitus? a. 102/60 mm Hg b. 128/76 mm Hg c. 139/90 mm Hg d. 136/82 mm Hg

ANS: B The goal for antihypertensive therapy for a patient with hypertension and diabetes mellitus is a BP <130/80 mm Hg. The BP of 102/60 may indicate overtreatment of the hypertension and an increased risk for adverse drug effects. The other two blood pressures indicate a need for modifications in the patient's treatment

A patient's complete blood count (CBC) shows a hemoglobin of 19 g/dL and a hematocrit of 54%. Which question should the nurse ask to determine possible causes of this finding? a. "Have you had a recent weight loss?" b. "Do you have any history of lung disease?" c. "Have you noticed any dark or bloody stools?" d. "What is your dietary intake of meats and protein?"

ANS: B The hemoglobin and hematocrit results indicate polycythemia, which can be associated with chronic obstructive pulmonary disease (COPD). The other questions would be appropriate for patients who are anemic

When developing a community health program to decrease the incidence of rheumatic fever, which action would be most important for the community health nurse to include? a. Vaccinate high-risk groups in the community with streptococcal vaccine. b. Teach community members to seek treatment for streptococcal pharyngitis. c. Teach about the importance of monitoring temperature when sore throats occur. d. Teach about prophylactic antibiotics to those with a family history of rheumatic fever.

ANS: B The incidence of rheumatic fever is decreased by treatment of streptococcal infections with antibiotics. Family history is not a risk factor for rheumatic fever. There is no immunization that is effective in decreasing the incidence of rheumatic fever. Teaching about monitoring temperature will not decrease the incidence of rheumatic fever.

An older patient is being discharged from the ambulatory surgical unit following left eye surgery. The patient tells the nurse, "I do not know if I can take care of myself with this patch over my eye." Which action by the nurse is most appropriate? a. Refer the patient for home health care services. b. Discuss the specific concerns regarding self-care. c. Give the patient written instructions regarding care. d. Assess the patient's support system for care at home.

ANS: B The nurse's initial action should be to assess exactly the patient's concerns about self-care. Referral to home health care and assessment of the patient's support system may be appropriate actions but will be based on further assessment of the patient's concerns. Written instructions should be given to the patient, but these are unlikely to address the patient's stated concern about self-care

An older patient who had knee replacement surgery 2 days ago can only tolerate being out of bed with physical therapy twice a day. Which collaborative problem should the nurse identify as a priority for this patient? a. Potential complication: hypovolemic shock b. Potential complication: venous thromboembolism c. Potential complication: fluid and electrolyte imbalance d. Potential complication: impaired surgical wound healing

ANS: B The patient is older and relatively immobile, which are two risk factors for development of deep vein thrombosis. The other potential complications are possible postoperative problems, but they are not supported by the data about this patient

A 30-year-old man with acute myelogenous leukemia develops an absolute neutrophil count of 850/µL while receiving outpatient chemotherapy. Which action by the outpatient clinic nurse is most appropriate? a. Discuss the need for hospital admission to treat the neutropenia. b. Teach the patient to administer filgrastim (Neupogen) injections. c. Plan to discontinue the chemotherapy until the neutropenia resolves. d. Order a high-efficiency particulate air (HEPA) filter for the patient's home.

ANS: B The patient may be taught to self-administer filgrastim injections. Although chemotherapy may be stopped with severe neutropenia (neutrophil count less than 500/µL), administration of filgrastim usually allows the chemotherapy to continue. Patients with neutropenia are at higher risk for infection when exposed to other patients in the hospital. HEPA filters are expensive and are used in the hospital, where the number of pathogens is much higher than in the patient's home environment

Which action will the nurse in the hypertension clinic take in order to obtain an accurate baseline blood pressure (BP) for a new patient? a. Deflate the BP cuff at a rate of 5 to 10 mm Hg per second. b. Have the patient sit in a chair with the feet flat on the floor. c. Assist the patient to the supine position for BP measurements. d. Obtain two BP readings in the dominant arm and average the results.

ANS: B The patient should be seated with the feet flat on the floor. The BP is obtained in both arms, and the results of the two arms are not averaged. The patient does not need to be in the supine position. The cuff should be deflated at 2 to 3 mm Hg per second

After receiving the following information about four patients during change-of-shift report, which patient should the nurse assess first? a. Patient with acute pericarditis who has a pericardial friction rub b. Patient who has just returned to the unit after balloon valvuloplasty c. Patient who has hypertrophic cardiomyopathy and a heart rate of 116 d. Patient with a mitral valve replacement who has an anticoagulant scheduled

ANS: B The patient who has just arrived after balloon valvuloplasty will need assessment for complications such as bleeding and hypotension. The information about the other patients is consistent with their diagnoses and does not indicate any complications or need for urgent assessment or intervention. DIF

A transesophageal echocardiogram (TEE) is ordered for a patient with possible endocarditis. Which action included in the standard TEE orders will the nurse need to accomplish first? a. Start an IV line. b. Place the patient on NPO status. c. Administer O2 per nasal cannula. d. Give lorazepam (Ativan) 1 mg IV.

ANS: B The patient will need to be NPO for 6 hours preceding the TEE, so the nurse should place the patient on NPO status as soon as the order is received. The other actions also will need to be accomplished but not until just before or during the procedure

Several patients call the outpatient clinic and ask to make an appointment as soon as possible. Which patient should the nurse schedule to be seen first? a. 44-year-old with sickle cell anemia who says "my eyes always look sort of yellow" b. 23-year-old with no previous health problems who has a nontender lump in the axilla c. 50-year-old with early-stage chronic lymphocytic leukemia who reports chronic fatigue d. 19-year-old with hemophilia who wants to learn to self-administer factor VII replacement

ANS: B The patient's age and presence of a nontender axillary lump suggest possible lymphoma, which needs rapid diagnosis and treatment. The other patients have questions about treatment or symptoms that are consistent with their diagnosis but do not need to be seen urgently

In the postanesthesia care unit (PACU), a patient's vital signs are blood pressure 116/72, pulse 74, respirations 12, and SpO2 91%. The patient is sleepy but awakens easily. Which action should the nurse take first? a. Place the patient in a side-lying position. b. Encourage the patient to take deep breaths. c. Prepare to transfer the patient to a clinical unit. d. Increase the rate of the postoperative IV fluids.

ANS: B The patient's borderline SpO2 and sleepiness indicate hypoventilation. The nurse should stimulate the patient and remind the patient to take deep breaths. Placing the patient in a lateral position is needed when the patient first arrives in the PACU and is unconscious. The stable blood pressure and pulse indicate that no changes in fluid intake are required. The patient is not fully awake and has a low SpO2, indicating that transfer from the PACU to a clinical unit is not appropriate

A patient whose heart monitor shows sinus tachycardia, rate 132, is apneic and has no palpable pulses. What is the first action that the nurse should take? a. Perform synchronized cardioversion. b. Start cardiopulmonary resuscitation (CPR). c. Administer atropine per agency dysrhythmia protocol. d. Provide supplemental oxygen via non-rebreather mask.

ANS: B The patient's clinical manifestations indicate pulseless electrical activity and the nurse should immediately start CPR. The other actions would not be of benefit to this patient

The nurse is assessing a patient who has been admitted to the intensive care unit (ICU) with a hypertensive emergency. Which finding is most important to report to the health care provider? a. Urine output over 8 hours is 250 mL less than the fluid intake. b. The patient cannot move the left arm and leg when asked to do so. c. Tremors are noted in the fingers when the patient extends the arms. d. The patient complains of a headache with pain at level 8/10 (0 to 10 scale).

ANS: B The patient's inability to move the left arm and leg indicates that a hemorrhagic stroke may be occurring and will require immediate action to prevent further neurologic damage. The other clinical manifestations are also likely caused by the hypertension and will require rapid nursing actions, but they do not require action as urgently as the neurologic changes

A patient with rheumatic fever has subcutaneous nodules, erythema marginatum, and polyarthritis. Based on these findings, which nursing diagnosis would be most appropriate? a. Pain related to permanent joint fixation b. Activity intolerance related to arthralgia c. Risk for infection related to open skin lesions d. Risk for impaired skin integrity related to pruritus

ANS: B The patient's joint pain will lead to difficulty with activity. The skin lesions seen in rheumatic fever are not open or pruritic. Although acute joint pain will be a problem for this patient, joint inflammation is a temporary clinical manifestation of rheumatic fever and is not associated with permanent joint changes

After receiving change-of-shift report on a heart failure unit, which patient should the nurse assess first? a. Patient who is taking carvedilol (Coreg) and has a heart rate of 58 b. Patient who is taking digoxin and has a potassium level of 3.1 mEq/L c. Patient who is taking isosorbide dinitrate/hydralazine (BiDil) and has a headache d. Patient who is taking captopril (Capoten) and has a frequent nonproductive cough

ANS: B The patient's low potassium level increases the risk for digoxin toxicity and potentially fatal dysrhythmias. The nurse should assess the patient for other signs of digoxin toxicity and then notify the health care provider about the potassium level. The other patients also have side effects of their medications, but their symptoms do not indicate potentially life-threatening complications

A patient with septicemia develops prolonged bleeding from venipuncture sites and blood in the stools. Which action is most important for the nurse to take? a. Avoid venipunctures. b. Notify the patient's physician. c. Apply sterile dressings to the sites. d. Give prescribed proton-pump inhibitors.

ANS: B The patient's new onset of bleeding and diagnosis of sepsis suggest that disseminated intravascular coagulation (DIC) may have developed, which will require collaborative actions such as diagnostic testing, blood product administration, and heparin administration. The other actions also are appropriate, but the most important action should be to notify the physician so that DIC treatment can be initiated rapidly

A patient with terminal cancer-related pain and a history of opioid abuse complains of breakthrough pain 2 hours before the next dose of sustained-release morphine sulfate (MS Contin) is due. Which action should the nurse take first? a. Use distraction by talking about things the patient enjoys. b. Administer the prescribed PRN immediate-acting morphine. c. Suggest the use of alternative therapies such as heat or cold. d. Consult with the doctor about increasing the MS Contin dose.

ANS: B The patient's pain requires rapid treatment and the nurse should administer the immediate-acting morphine. Increasing the MS Contin dose and use of alternative therapies may also be needed, but the initial action should be to use the prescribed analgesic medications

The nurse obtains the following information from a patient newly diagnosed with prehypertension. Which finding is most important to address with the patient? a. Low dietary fiber intake b. No regular aerobic exercise c. Weight 5 pounds above ideal weight d. Drinks a beer with dinner on most nights

ANS: B The recommendations for preventing hypertension include exercising aerobically for 30 minutes most days of the week. A weight that is 5 pounds over the ideal body weight is not a risk factor for hypertension. The Dietary Approaches to Stop Hypertension (DASH) diet is high in fiber, but increasing fiber alone will not prevent hypertension from developing. The patient's alcohol intake is within guidelines and will not increase the hypertension risk

Which action could the postanesthesia care unit (PACU) nurse delegate to unlicensed assistive personnel (UAP) who help with the transfer of a patient to the clinical unit? a. Clarify the postoperative orders with the surgeon. b. Help with the transfer of the patient onto a stretcher. c. Document the appearance of the patient's incision in the chart. d. Provide hand off communication to the surgical unit charge nurse.

ANS: B The scope of practice of UAP includes repositioning and moving patients under the supervision of a nurse. Providing report to another nurse, assessing and documenting the wound appearance, and clarifying physician orders with another nurse require registered-nurse (RN) level education and scope of practice

A 68-year-old woman with acute myelogenous leukemia (AML) asks the nurse whether the planned chemotherapy will be worth undergoing. Which response by the nurse is appropriate? a. "If you do not want to have chemotherapy, other treatment options include stem cell transplantation." b. "The side effects of chemotherapy are difficult, but AML frequently goes into remission with chemotherapy." c. "The decision about treatment is one that you and the doctor need to make rather than asking what I would do." d. "You don't need to make a decision about treatment right now because leukemias in adults tend to progress quite slowly."

ANS: B This response uses therapeutic communication by addressing the patient's question and giving accurate information. The other responses either give inaccurate information or fail to address the patient's question, which will discourage the patient from asking the nurse for information

When planning care for a patient hospitalized with a streptococcal infective endocarditis (IE), which intervention is a priority for the nurse to include? a. Monitor labs for streptococcal antibodies. b. Arrange for placement of a long-term IV catheter. c. Teach the importance of completing all oral antibiotics. d. Encourage the patient to begin regular aerobic exercise.

ANS: B Treatment for IE involves 4 to 6 weeks of IV antibiotic therapy in order to eradicate the bacteria, which will require a long-term IV catheter such as a peripherally inserted central catheter (PICC) line. Rest periods and limiting physical activity to a moderate level are recommended during the treatment for IE. Oral antibiotics are not effective in eradicating the infective bacteria that cause IE. Blood cultures, rather than antibody levels, are used to monitor the effectiveness of antibiotic therapy

Which admission order written by the health care provider for a patient admitted with infective endocarditis (IE) and a fever would be a priority for the nurse to implement? a. Administer ceftriaxone (Rocephin) 1 g IV. b. Order blood cultures drawn from two sites. c. Give acetaminophen (Tylenol) PRN for fever. d. Arrange for a transesophageal echocardiogram.

ANS: B Treatment of the IE with antibiotics should be started as quickly as possible, but it is essential to obtain blood cultures before initiating antibiotic therapy to obtain accurate sensitivity results. The echocardiogram and acetaminophen administration also should be implemented rapidly, but the blood cultures (and then administration of the antibiotic) have the highest priority

The nurse and unlicensed assistive personnel (UAP) on the telemetry unit are caring for four patients. Which nursing action can be delegated to the UAP? a. Teaching a patient scheduled for exercise electrocardiography about the procedure b. Placing electrodes in the correct position for a patient who is to receive ECG monitoring c. Checking the catheter insertion site for a patient who is recovering from a coronary angiogram d. Monitoring a patient who has just returned to the unit after a transesophageal echocardiogram

ANS: B UAP can be educated in standardized lead placement for ECG monitoring. Assessment of patients who have had procedures where airway maintenance (transesophageal echocardiography) or bleeding (coronary angiogram) is a concern must be done by the registered nurse (RN). Patient teaching requires RN level education and scope of practice

The nurse is planning to administer a transfusion of packed red blood cells (PRBCs) to a patient with blood loss from gastrointestinal hemorrhage. Which action can the nurse delegate to unlicensed assistive personnel (UAP)? a. Verify the patient identification (ID) according to hospital policy. b. Obtain the temperature, blood pressure, and pulse before the transfusion. c. Double-check the product numbers on the PRBCs with the patient ID band. d. Monitor the patient for shortness of breath or chest pain during the transfusion.

ANS: B UAP education includes measurement of vital signs. UAP would report the vital signs to the registered nurse (RN). The other actions require more education and a larger scope of practice and should be done by licensed nursing staff members

The nurse notes that a patient's cardiac monitor shows that every other beat is earlier than expected, has no visible P wave, and has a QRS complex that is wide and bizarre in shape. How will the nurse document the rhythm? a. Ventricular couplets b. Ventricular bigeminy c. Ventricular R-on-T phenomenon d. Multifocal premature ventricular contractions

ANS: B Ventricular bigeminy describes a rhythm in which every other QRS complex is wide and bizarre looking. Pairs of wide QRS complexes are described as ventricular couplets. There is no indication that the premature ventricular contractions (PVCs) are multifocal or that the R-on-T phenomenon is occurring

Which action should the nurse perform when preparing a patient with supraventricular tachycardia for cardioversion who is alert and has a blood pressure of 110/66 mm Hg? a. Turn the synchronizer switch to the "off" position. b. Give a sedative before cardioversion is implemented. c. Set the defibrillator/cardioverter energy to 360 joules. d. Provide assisted ventilations with a bag-valve-mask device.

ANS: B When a patient has a nonemergency cardioversion, sedation is used just before the procedure. The synchronizer switch is turned "on" for cardioversion. The initial level of joules for cardioversion is low (e.g., 50). Assisted ventilations are not indicated for this patient

The health care provider orders a patient-controlled analgesia (PCA) machine to provide pain relief for a patient with acute surgical pain who has never received opioids in the past. Which nursing actions regarding opioid administration are appropriate at this time (select all that apply)? a. Assess for signs that the patient is becoming addicted to the opioid. b. Monitor for therapeutic and adverse effects of opioid administration. c. Emphasize that the risk of some opioid side effects increases over time. d. Teach the patient about how analgesics improve postoperative activity levels. e. Provide instructions on decreasing opioid doses by the second postoperative day.

ANS: B, D Monitoring for pain relief and teaching the patient about how opioid use will improve postoperative outcomes are appropriate actions when administering opioids for acute pain. Although postoperative patients usually need a decreasing amount of opioids by the second postoperative day, each patient's response is individual. Tolerance may occur, but addiction to opioids will not develop in the acute postoperative period. The patient should use the opioids to achieve adequate pain control, and so the nurse should not emphasize the adverse effects

A patient's T-tube is draining dark green fluid after gallbladder surgery. What action by the nurse is the most appropriate? a. Notify the patient's surgeon. b. Place the patient on bed rest. c. Document the color and amount of drainage. d. Irrigate the T-tube with sterile normal saline.

ANS: C A T-tube normally drains dark green to bright yellow drainage, so no action other than to document the amount and color of the drainage is needed. The other actions are not necessary

The nurse reviews the medication orders for an older patient with arthritis in both hips who is complaining of level 3 (0 to 10 scale) hip pain while ambulating. Which medication should the nurse use as initial therapy? a. Naproxen (Aleve) 200 mg orally b. Oxycodone (Roxicodone) 5 mg orally c. Acetaminophen (Tylenol) 650 mg orally d. Aspirin (acetylsalicylic acid, ASA) 650 mg orally

ANS: C Acetaminophen is the best first-choice medication. The principle of "start low, go slow" is used to guide therapy when treating older adults because the ability to metabolize medications is decreased and the likelihood of medication interactions is increased. Nonopioid analgesics are used first for mild to moderate pain, although opioids may be used later. Aspirin and the NSAIDs are associated with a high incidence of gastrointestinal bleeding in older patients

Which action for a patient with neutropenia is appropriate for the registered nurse (RN) to delegate to a licensed practical/vocational nurse (LPN/LVN)? a. Assessing the patient for signs and symptoms of infection b. Teaching the patient the purpose of neutropenic precautions c. Administering subcutaneous filgrastim (Neupogen) injection d. Developing a discharge teaching plan for the patient and family

ANS: C Administration of subcutaneous medications is included in LPN/LVN education and scope of practice. Patient education, assessment, and developing the plan of care require RN level education and scope of practice

The nasogastric (NG) tube is removed on the second postoperative day, and the patient is placed on a clear liquid diet. Four hours later, the patient complains of sharp, cramping gas pains. What action by the nurse is the most appropriate? a. Reinsert the NG tube. b. Give the PRN IV opioid. c. Assist the patient to ambulate. d. Place the patient on NPO status.

ANS: C Ambulation encourages peristalsis and the passing of flatus, which will relieve the patient's discomfort. If distention persists, the patient may need to be placed on NPO status, but usually this is not necessary. Morphine administration will further decrease intestinal motility. Gas pains are usually caused by trapping of flatus in the colon, and reinsertion of the NG tube will not relieve the pains

A patient who is receiving dobutamine (Dobutrex) for the treatment of acute decompensated heart failure (ADHF) has the following nursing interventions included in the plan of care. Which action will be most appropriate for the registered nurse (RN) to delegate to an experienced licensed practical/vocational nurse (LPN/LVN)? a. Assess the IV insertion site for signs of extravasation. b. Teach the patient the reasons for remaining on bed rest. c. Monitor the patient's blood pressure and heart rate every hour. d. Titrate the rate to keep the systolic blood pressure >90 mm Hg.

ANS: C An experienced LPN/LVN would be able to monitor BP and heart rate and would know to report significant changes to the RN. Teaching patients, making adjustments to the drip rate for vasoactive medications, and monitoring for serious complications such as extravasation require RN level education and scope of practice

During discharge teaching with a 68-year-old patient who had a mitral valve replacement with a mechanical valve, the nurse instructs the patient on the a. use of daily aspirin for anticoagulation. b. correct method for taking the radial pulse. c. need for frequent laboratory blood testing. d. need to avoid any physical activity for 1 month.

ANS: C Anticoagulation with warfarin (Coumadin) is needed for a patient with mechanical valves to prevent clotting on the valve. This will require frequent international normalized ratio (INR) testing. Daily aspirin use will not be effective in reducing the risk for clots on the valve. Monitoring of the radial pulse is not necessary after valve replacement. Patients should resume activities of daily living as tolerated

When a patient with splenomegaly is scheduled for splenectomy, which action will the nurse include in the preoperative plan of care? a. Discourage deep breathing to reduce risk for splenic rupture. b. Teach the patient to use ibuprofen (Advil) for left upper quadrant pain. c. Schedule immunization with the pneumococcal vaccine (Pneumovax). d. Avoid the use of acetaminophen (Tylenol) for 2 weeks prior to surgery.

ANS: C Asplenic patients are at high risk for infection with Pneumococcus and immunization reduces this risk. There is no need to avoid acetaminophen use before surgery, but nonsteroidal antiinflammatory drugs (NSAIDs) may increase bleeding risk and should be avoided. The enlarged spleen may decrease respiratory depth and the patient should be encouraged to take deep breaths

Which diagnostic test will be most useful to the nurse in determining whether a patient admitted with acute shortness of breath has heart failure? a. Serum troponin b. Arterial blood gases c. B-type natriuretic peptide d. 12-lead electrocardiogram

ANS: C B-type natriuretic peptide (BNP) is secreted when ventricular pressures increase, as they do with heart failure. Elevated BNP indicates a probable or very probable diagnosis of heart failure. A twelve-lead electrocardiogram, arterial blood gases, and troponin may also be used in determining the causes or effects of heart failure but are not as clearly diagnostic of heart failure as BNP

A 52-year-old patient has a new diagnosis of pernicious anemia. The nurse determines that the patient understands the teaching about the disorder when the patient states, "I a. need to start eating more red meat and liver." b. will stop having a glass of wine with dinner." c. could choose nasal spray rather than injections of vitamin B12." d. will need to take a proton pump inhibitor like omeprazole (Prilosec)."

ANS: C Because pernicious anemia prevents the absorption of vitamin B12, this patient requires injections or intranasal administration of cobalamin. Alcohol use does not cause cobalamin deficiency. Proton pump inhibitors decrease the absorption of vitamin B12. Eating more foods rich in vitamin B12 is not helpful because the lack of intrinsic factor prevents absorption of the vitamin

Which action by the nurse will determine if the therapies ordered for a patient with chronic constrictive pericarditis are effective? a. Assess for the presence of a paradoxical pulse. b. Monitor for changes in the patient's sedimentation rate. c. Assess for the presence of jugular venous distention (JVD). d. Check the electrocardiogram (ECG) for ST segment changes.

ANS: C Because the most common finding on physical examination for a patient with chronic constrictive pericarditis is jugular venous distention, a decrease in JVD indicates improvement. Paradoxical pulse, ST-segment ECG changes, and changes in sedimentation rates occur with acute pericarditis but are not expected in chronic constrictive pericarditis

A patient is transferred from the postanesthesia care unit (PACU) to the clinical unit. Which action by the nurse on the clinical unit should be performed first? a. Assess the patient's pain. b. Orient the patient to the unit. c. Take the patient's vital signs. d. Read the postoperative orders.

ANS: C Because the priority concerns after surgery are airway, breathing, and circulation, the vital signs are assessed first. The other actions should take place after the vital signs are obtained and compared with the vital signs before transfer

A patient who is on the progressive care unit develops atrial flutter, rate 150, with associated dyspnea and chest pain. Which action that is included in the hospital dysrhythmia protocol should the nurse do first? a. Obtain a 12-lead electrocardiogram (ECG). b. Notify the health care provider of the change in rhythm. c. Give supplemental O2 at 2 to 3 L/min via nasal cannula. d. Assess the patient's vital signs including oxygen saturation.

ANS: C Because this patient has dyspnea and chest pain in association with the new rhythm, the nurse's initial actions should be to address the patient's airway, breathing, and circulation (ABC) by starting with oxygen administration. The other actions also are important and should be implemented rapidly

A patient with heart failure has a new order for captopril (Capoten) 12.5 mg PO. After administering the first dose and teaching the patient about the drug, which statement by the patient indicates that teaching has been effective? a. "I will be sure to take the medication with food." b. "I will need to eat more potassium-rich foods in my diet." c. "I will call for help when I need to get up to use the bathroom." d. "I will expect to feel more short of breath for the next few days."

ANS: C Captopril can cause hypotension, especially after the initial dose, so it is important that the patient not get up out of bed without assistance until the nurse has had a chance to evaluate the effect of the first dose. The angiotensin-converting enzyme (ACE) inhibitors are potassium sparing, and the nurse should not teach the patient to purposely increase sources of dietary potassium. Increased shortness of breath is expected with the initiation of b-adrenergic blocker therapy for heart failure, not for ACE inhibitor therapy. ACE inhibitors are best absorbed when taken an hour before eating

The nurse has received the laboratory results for a patient who developed chest pain 4 hours ago and may be having a myocardial infarction. The most important laboratory result to review will be a. myoglobin. b. low-density lipoprotein (LDL) cholesterol. c. troponins T and I. d. creatine kinase-MB (CK-MB).

ANS: C Cardiac troponins start to elevate 4 to 6 hours after myocardial injury and are highly specific to myocardium. They are the preferred diagnostic marker for myocardial infarction. Myoglobin rises in response to myocardial injury within 30 to 60 minutes. It is rapidly cleared from the body, thus limiting its use in the diagnosis of myocardial infarction. LDL cholesterol is useful in assessing cardiovascular risk but is not helpful in determining whether a patient is having an acute myocardial infarction. Creatine kinase (CK-MB) is specific to myocardial injury and infarction and increases 4 to 6 hours after the infarction occurs. It is often trended with troponin levels

The nurse will plan discharge teaching about the need for prophylactic antibiotics when having dental procedures for which patient? a. Patient admitted with a large acute myocardial infarction. b. Patient being discharged after an exacerbation of heart failure. c. Patient who had a mitral valve replacement with a mechanical valve. d. Patient being treated for rheumatic fever after a streptococcal infection.

ANS: C Current American Heart Association guidelines recommend the use of prophylactic antibiotics before dental procedures for patients with prosthetic valves to prevent infective endocarditis (IE). The other patients are not at risk for IE

When caring for a patient with infective endocarditis of the tricuspid valve, the nurse should monitor the patient for the development of a. flank pain. b. splenomegaly. c. shortness of breath. d. mental status changes.

ANS: C Embolization from the tricuspid valve would cause symptoms of pulmonary embolus. Flank pain, changes in mental status, and splenomegaly would be associated with embolization from the left-sided valves

Which instruction will the nurse plan to include in discharge teaching for the patient admitted with a sickle cell crisis? a. Take a daily multivitamin with iron. b. Limit fluids to 2 to 3 quarts per day. c. Avoid exposure to crowds when possible. d. Drink only two caffeinated beverages daily.

ANS: C Exposure to crowds increases the patient's risk for infection, the most common cause of sickle cell crisis. There is no restriction on caffeine use. Iron supplementation is generally not recommended. A high-fluid intake is recommended

The nurse assesses a patient with pernicious anemia. Which assessment finding would the nurse expect? a. Yellow-tinged sclerae b. Shiny, smooth tongue c. Numbness of the extremities d. Gum bleeding and tenderness

ANS: C Extremity numbness is associated with cobalamin (vitamin B12) deficiency or pernicious anemia. Loss of the papillae of the tongue occurs with chronic iron deficiency. Yellow-tinged sclera is associated with hemolytic anemia and the resulting jaundice. Gum bleeding and tenderness occur with thrombocytopenia or neutropenia

An older patient has been diagnosed with possible white coat hypertension. Which action will the nurse plan to take next? a. Schedule the patient for regular blood pressure (BP) checks in the clinic. b. Instruct the patient about the need to decrease stress levels. c. Tell the patient how to self-monitor and record BPs at home. d. Inform the patient that ambulatory blood pressure monitoring will be needed.

ANS: C Having the patient self-monitor BPs at home will provide a reliable indication about whether the patient has hypertension. Regular BP checks in the clinic are likely to be high in a patient with white coat hypertension. Ambulatory blood pressure monitoring may be used if the data from self-monitoring are unclear. Although elevated stress levels may contribute to hypertension, instructing the patient about this is unlikely to reduce BP

A patient who had a total hip replacement had an intraoperative hemorrhage 14 hours ago. Which laboratory result would the nurse expect to find? a. Hematocrit of 46% b. Hemoglobin of 13.8 g/dL c. Elevated reticulocyte count d. Decreased white blood cell (WBC) count

ANS: C Hemorrhage causes the release of reticulocytes (immature red blood cells) from the bone marrow into circulation. The hematocrit and hemoglobin levels are normal. The WBC count is not affected by bleeding

A patient has recently started on digoxin (Lanoxin) in addition to furosemide (Lasix) and captopril (Capoten) for the management of heart failure. Which assessment finding by the home health nurse is a priority to communicate to the health care provider? a. Presence of 1 to 2+ edema in the feet and ankles b. Palpable liver edge 2 cm below the ribs on the right side c. Serum potassium level 3.0 mEq/L after 1 week of therapy d. Weight increase from 120 pounds to 122 pounds over 3 days

ANS: C Hypokalemia can predispose the patient to life-threatening dysrhythmias (e.g., premature ventricular contractions), and potentiate the actions of digoxin and increase the risk for digoxin toxicity, which can also cause life-threatening dysrhythmias. The other data indicate that the patient's heart failure requires more effective therapies, but they do not require nursing action as rapidly as the low serum potassium level

A patient has a junctional escape rhythm on the monitor. The nurse will expect the patient to have a heart rate of _____ beats/minute. a. 15 to 20 b. 20 to 40 c. 40 to 60 d. 60 to 100

ANS: C If the sinoatrial (SA) node fails to discharge, the atrioventricular (AV) node will automatically discharge at the normal rate of 40 to 60 beats/minute. The slower rates are typical of the bundle of His and the Purkinje system and may be seen with failure of both the SA and AV node to discharge. The normal SA node rate is 60 to 100 beats/minute

The registered nurse (RN) is caring for a patient with a hypertensive crisis who is receiving sodium nitroprusside (Nipride). Which nursing action can the nurse delegate to an experienced licensed practical/vocational nurse (LPN/LVN)? a. Titrate nitroprusside to decrease mean arterial pressure (MAP) to 115 mm Hg. b. Evaluate effectiveness of nitroprusside therapy on blood pressure (BP). c. Set up the automatic blood pressure machine to take BP every 15 minutes. d. Assess the patient's environment for adverse stimuli that might increase BP.

ANS: C LPN/LVN education and scope of practice include the correct use of common equipment such as automatic blood pressure machines. The other actions require advanced nursing judgment and education, and should be done by RNs

Which action should the nurse take when administering the initial dose of oral labetalol (Normodyne) to a patient with hypertension? a. Encourage the use of hard candy to prevent dry mouth. b. Instruct the patient to ask for help if heart palpitations occur. c. Ask the patient to request assistance when getting out of bed. d. Teach the patient that headaches may occur with this medication.

ANS: C Labetalol decreases sympathetic nervous system activity by blocking both á- and b-adrenergic receptors, leading to vasodilation and a decrease in heart rate, which can cause severe orthostatic hypotension. Heart palpitations, dry mouth, dehydration, and headaches are possible side effects of other antihypertensives

A 21-year-old woman is scheduled for percutaneous transluminal balloon valvuloplasty to treat mitral stenosis. Which information should the nurse include when explaining the advantages of valvuloplasty over valve replacement to the patient? a. Biologic valves will require immunosuppressive drugs after surgery. b. Mechanical mitral valves need to be replaced sooner than biologic valves. c. Lifelong anticoagulant therapy will be needed after mechanical valve replacement. d. Ongoing cardiac care by a health care provider is not necessary after valvuloplasty.

ANS: C Long-term anticoagulation therapy is needed after mechanical valve replacement, and this would restrict decisions about career and childbearing in this patient. Mechanical valves are durable and last longer than biologic valves. All valve repair procedures are palliative, not curative, and require lifelong health care. Biologic valves do not activate the immune system, and immunosuppressive therapy is not needed

Which information obtained by the nurse who is admitting the patient for magnetic resonance imaging (MRI) will be most important to report to the health care provider before the MRI? a. The patient has an allergy to shellfish. b. The patient has a history of atherosclerosis. c. The patient has a permanent ventricular pacemaker. d. The patient took all the prescribed cardiac medications today.

ANS: C MRI is contraindicated for patients with implanted metallic devices such as pacemakers. The other information also will be reported to the health care provider but does not impact on whether or not the patient can have an MRI

The nurse plans discharge teaching for a patient with chronic heart failure who has prescriptions for digoxin (Lanoxin) and hydrochlorothiazide (HydroDIURIL). Appropriate instructions for the patient include a. limit dietary sources of potassium. b. take the hydrochlorothiazide before bedtime. c. notify the health care provider if nausea develops. d. skip the digoxin if the pulse is below 60 beats/minute.

ANS: C Nausea is an indication of digoxin toxicity and should be reported so that the provider can assess the patient for toxicity and adjust the digoxin dose, if necessary. The patient will need to include potassium-containing foods in the diet to avoid hypokalemia. Patients should be taught to check their pulse daily before taking the digoxin and if the pulse is less than 60, to call their provider before taking the digoxin. Diuretics should be taken early in the day to avoid sleep disruption

The nurse is caring for a patient the first postoperative day following a laparotomy for a small bowel obstruction. The nurse notices new bright-red drainage about 5 cm in diameter on the dressing. Which action should the nurse take first? a. Reinforce the dressing. b. Apply an abdominal binder. c. Take the patient's vital signs. d. Recheck the dressing in 1 hour for increased drainage.

ANS: C New bright-red drainage may indicate hemorrhage, and the nurse should initially assess the patient's vital signs for tachycardia and hypotension. The surgeon should then be notified of the drainage and the vital signs. The dressing may be changed or reinforced, based on the surgeon's orders or institutional policy. The nurse should not wait an hour to recheck the dressing

The nurse teaches a student nurse about the action of ibuprofen. Which statement, if made by the student, indicates that teaching was effective? a. "The drug decreases pain impulses in the spinal cord." b. "The drug decreases sensitivity of the brain to painful stimuli." c. "The drug decreases production of pain-sensitizing chemicals." d. "The drug decreases the modulating effect of descending nerves."

ANS: C Nonsteroidal antiinflammatory drugs (NSAIDs) provide analgesic effects by decreasing the production of pain-sensitizing chemicals such as prostaglandins at the site of injury. Transmission of impulses through the spinal cord, brain sensitivity to pain, and the descending nerve pathways are not affected by NSAIDs

A 54-year-old woman with acute myelogenous leukemia (AML) is considering treatment with a hematopoietic stem cell transplant (HSCT). The best approach for the nurse to assist the patient with a treatment decision is to a. emphasize the positive outcomes of a bone marrow transplant. b. discuss the need for adequate insurance to cover post-HSCT care. c. ask the patient whether there are any questions or concerns about HSCT. d. explain that a cure is not possible with any other treatment except HSCT.

ANS: C Offering the patient an opportunity to ask questions or discuss concerns about HSCT will encourage the patient to voice concerns about this treatment and also will allow the nurse to assess whether the patient needs more information about the procedure. Treatment of AML using chemotherapy is another option for the patient. It is not appropriate for the nurse to ask the patient to consider insurance needs in making this decision

A postoperative patient has a nursing diagnosis of ineffective airway clearance. The nurse determines that interventions for this nursing diagnosis have been successful if which is observed? a. Patient drinks 2 to 3 L of fluid in 24 hours. b. Patient uses the spirometer 10 times every hour. c. Patient's breath sounds are clear to auscultation. d. Patient's temperature is less than 100.4° F orally.

ANS: C One characteristic of ineffective airway clearance is the presence of adventitious breath sounds such as rhonchi or crackles, so clear breath sounds are an indication of resolution of the problem. Spirometer use and increased fluid intake are interventions for ineffective airway clearance but may not improve breath sounds in all patients. Elevated temperature may occur with atelectasis, but a normal or near-normal temperature does not always indicate resolution of respiratory problems

A patient who has chronic heart failure tells the nurse, "I was fine when I went to bed, but I woke up in the middle of the night feeling like I was suffocating!" The nurse will document this assessment finding as a. orthopnea. b. pulsus alternans. c. paroxysmal nocturnal dyspnea. d. acute bilateral pleural effusion.

ANS: C Paroxysmal nocturnal dyspnea is caused by the reabsorption of fluid from dependent body areas when the patient is sleeping and is characterized by waking up suddenly with the feeling of suffocation. Pulsus alternans is the alternation of strong and weak peripheral pulses during palpation. Orthopnea indicates that the patient is unable to lie flat because of dyspnea. Pleural effusions develop over a longer time period

Which statement by a patient with restrictive cardiomyopathy indicates that the nurse's discharge teaching about self-management has been most effective? a. "I will avoid taking aspirin or other antiinflammatory drugs." b. "I will need to limit my intake of salt and fluids even in hot weather." c. "I will take antibiotics when my teeth are cleaned at the dental office." d. "I should begin an exercise program that includes things like biking or swimming."

ANS: C Patients with restrictive cardiomyopathy are at risk for infective endocarditis and should use prophylactic antibiotics for any procedure that may cause bacteremia. The other statements indicate a need for more teaching by the nurse. Dehydration and vigorous exercise impair ventricular filling in patients with restrictive cardiomyopathy. There is no need to avoid salt (unless ordered), aspirin, or NSAIDs

Which information will the nurse include when teaching a patient who is scheduled for a radiofrequency catheter ablation for treatment of atrial flutter? a. The procedure will prevent or minimize the risk for sudden cardiac death. b. The procedure will use cold therapy to stop the formation of the flutter waves. c. The procedure will use electrical energy to destroy areas of the conduction system. d. The procedure will stimulate the growth of new conduction pathways between the atria.

ANS: C Radiofrequency catheter ablation therapy uses electrical energy to "burn" or ablate areas of the conduction system as definitive treatment of atrial flutter (i.e., restore normal sinus rhythm) and tachydysrhythmias. All other statements regarding the procedure are incorrect

IV sodium nitroprusside (Nipride) is ordered for a patient with acute pulmonary edema. During the first hours of administration, the nurse will need to titrate the nitroprusside rate if the patient develops a. ventricular ectopy. b. a dry, hacking cough. c. a systolic BP <90 mm Hg. d. a heart rate <50 beats/minute.

ANS: C Sodium nitroprusside is a potent vasodilator, and the major adverse effect is severe hypotension. Coughing and bradycardia are not adverse effects of this medication. Nitroprusside does not cause increased ventricular ectopy

The nurse working on the heart failure unit knows that teaching an older female patient with newly diagnosed heart failure is effective when the patient states that a. she will take furosemide (Lasix) every day at bedtime. b. the nitroglycerin patch is applied when any chest pain develops. c. she will call the clinic if her weight goes from 124 to 128 pounds in a week. d. an additional pillow can help her sleep if she is feeling short of breath at night.

ANS: C Teaching for a patient with heart failure includes information about the need to weigh daily and notify the health care provider about an increase of 3 pounds in 2 days or 3 to 5 pounds in a week. Nitroglycerin patches are used primarily to reduce preload (not to prevent chest pain) in patients with heart failure and should be used daily, not on an "as needed" basis. Diuretics should be taken earlier in the day to avoid nocturia and sleep disturbance. The patient should call the clinic if increased orthopnea develops, rather than just compensating by further elevating the head of the bed

A patient with chronic heart failure who is taking a diuretic and an angiotensin-converting enzyme (ACE) inhibitor and who is on a low-sodium diet tells the home health nurse about a 5-pound weight gain in the last 3 days. The nurse's priority action will be to a. have the patient recall the dietary intake for the last 3 days. b. ask the patient about the use of the prescribed medications. c. assess the patient for clinical manifestations of acute heart failure. d. teach the patient about the importance of restricting dietary sodium.

ANS: C The 5-pound weight gain over 3 days indicates that the patient's chronic heart failure may be worsening. It is important that the patient be assessed immediately for other clinical manifestations of decompensation, such as lung crackles. A dietary recall to detect hidden sodium in the diet, reinforcement of sodium restrictions, and assessment of medication compliance may be appropriate interventions but are not the first nursing actions indicated.

Which assessment finding obtained by the nurse when assessing a patient with acute pericarditis should be reported immediately to the health care provider? a. Pulsus paradoxus 8 mm Hg b. Blood pressure (BP) of 168/94 c. Jugular venous distention (JVD) to jaw level d. Level 6 (0 to 10 scale) chest pain with a deep breath

ANS: C The JVD indicates that the patient may have developed cardiac tamponade and may need rapid intervention to maintain adequate cardiac output. Hypertension would not be associated with complications of pericarditis, and the BP is not high enough to indicate that there is any immediate need to call the health care provider. A pulsus paradoxus of 8 mm Hg is normal. Level 6/10 chest pain should be treated but is not unusual with pericarditis

Which finding about a patient with polycythemia vera is most important for the nurse to report to the health care provider? a. Hematocrit 55% b. Presence of plethora c. Calf swelling and pain d. Platelet count 450,000/mL

ANS: C The calf swelling and pain suggest that the patient may have developed a deep vein thrombosis, which will require diagnosis and treatment to avoid complications such as pulmonary embolus. The other findings will also be reported to the health care provider but are expected in a patient with this diagnosis

Which topic will the nurse plan to include in discharge teaching for a patient with systolic heart failure and an ejection fraction of 33%? a. Need to begin an aerobic exercise program several times weekly b. Use of salt substitutes to replace table salt when cooking and at the table c. Benefits and side effects of angiotensin-converting enzyme (ACE) inhibitors d. Importance of making an annual appointment with the primary care provider

ANS: C The core measures for the treatment of heart failure established by The Joint Commission indicate that patients with an ejection fraction (EF) <40% receive an ACE inhibitor to decrease the progression of heart failure. Aerobic exercise may not be appropriate for a patient with this level of heart failure, salt substitutes are not usually recommended because of the risk of hyperkalemia, and the patient will need to see the primary care provider more frequently than annually

While admitting an 82-year-old with acute decompensated heart failure to the hospital, the nurse learns that the patient lives alone and sometimes confuses the "water pill" with the "heart pill." When planning for the patient's discharge the nurse will facilitate a a. consult with a psychologist. b. transfer to a long-term care facility. c. referral to a home health care agency. d. arrangements for around-the-clock care.

ANS: C The data about the patient suggest that assistance in developing a system for taking medications correctly at home is needed. A home health nurse will assess the patient's home situation and help the patient develop a method for taking the two medications as directed. There is no evidence that the patient requires services such as a psychologist consult, long-term care, or around-the-clock home care

The nurse is reviewing the laboratory results for newly admitted patients on the cardiovascular unit. Which patient laboratory result is most important to communicate as soon as possible to the health care provider? a. Patient whose triglyceride level is high b. Patient who has very low homocysteine level c. Patient with increase in troponin T and troponin I level d. Patient with elevated high-sensitivity C-reactive protein level

ANS: C The elevation in troponin T and I indicates that the patient has had an acute myocardial infarction. Further assessment and interventions are indicated. The other laboratory results are indicative of increased risk for coronary artery disease but are not associated with acute cardiac problems that need immediate intervention

Which action will the nurse include in the plan of care for a patient who has thalassemia major? a. Teach the patient to use iron supplements. b. Avoid the use of intramuscular injections. c. Administer iron chelation therapy as needed. d. Notify health care provider of hemoglobin 11g/dL.

ANS: C The frequent transfusions used to treat thalassemia major lead to iron toxicity in patients unless iron chelation therapy is consistently used. Iron supplementation is avoided in patients with thalassemia. There is no need to avoid intramuscular injections. The goal for patients with thalassemia major is to maintain a hemoglobin of 10 g/dL or greater

When visiting a hospice patient, the nurse assesses that the patient has a respiratory rate of 11 breaths/minute and complains of severe pain. Which action is best for the nurse to take? a. Inform the patient that increasing the morphine will cause the respiratory drive to fail. b. Tell the patient that additional morphine can be administered when the respirations are 12. c. Titrate the prescribed morphine dose upward until the patient indicates adequate pain relief. d. Administer a nonopioid analgesic, such as a nonsteroidal antiinflammatory drug (NSAID), to improve patient pain control.

ANS: C The goal of opioid use in terminally ill patients is effective pain relief regardless of adverse effects such as respiratory depression. A nonopioid analgesic like ibuprofen would not provide adequate analgesia or be absorbed quickly. The rule of double effect provides ethical justification for administering an increased morphine dose to provide effective pain control even though the morphine may further decrease the patient's respiratory rate.

While assessing a 68-year-old with ascites, the nurse also notes jugular venous distention (JVD) with the head of the patient's bed elevated 45 degrees. The nurse knows this finding indicates a. decreased fluid volume. b. jugular vein atherosclerosis. c. increased right atrial pressure. d. incompetent jugular vein valves.

ANS: C The jugular veins empty into the superior vena cava and then into the right atrium, so JVD with the patient sitting at a 45-degree angle reflects increased right atrial pressure. JVD is an indicator of excessive fluid volume (increased preload), not decreased fluid volume. JVD is not caused by incompetent jugular vein valves or atherosclerosis

During change-of-shift report, the nurse obtains the following information about a hypertensive patient who received the first dose of nadolol (Corgard) during the previous shift. Which information indicates that the patient needs immediate intervention? a. The patient's most recent blood pressure (BP) reading is 158/91 mm Hg. b. The patient's pulse has dropped from 68 to 57 beats/minute. c. The patient has developed wheezes throughout the lung fields. d. The patient complains that the fingers and toes feel quite cold.

ANS: C The most urgent concern for this patient is the wheezes, which indicate that bronchospasm (a common adverse effect of the noncardioselective b-blockers) is occurring. The nurse should immediately obtain an oxygen saturation measurement, apply supplemental oxygen, and notify the health care provider. The mild decrease in heart rate and complaint of cold fingers and toes are associated with b-receptor blockade but do not require any change in therapy. The BP reading may indicate that a change in medication type or dose may be indicated. However, this is not as urgently needed as addressing the bronchospasm

Which action will the nurse implement for a patient who arrives for a calcium-scoring CT scan? a. Insert an IV catheter. b. Administer oral sedative medications. c. Teach the patient about the procedure. d. Confirm that the patient has been fasting.

ANS: C The nurse will need to teach the patient that the procedure is rapid and involves little risk. None of the other actions are necessary

A patient recovering from heart surgery develops pericarditis and complains of level 6 (0 to 10 scale) chest pain with deep breathing. Which ordered PRN medication will be the most appropriate for the nurse to give? a. Fentanyl 1 mg IV b. IV morphine sulfate 4 mg c. Oral ibuprofen (Motrin) 600 mg d. Oral acetaminophen (Tylenol) 650 mg

ANS: C The pain associated with pericarditis is caused by inflammation, so nonsteroidal antiinflammatory drugs (NSAIDs) (e.g., ibuprofen) are most effective. Opioid analgesics are usually not used for the pain associated with pericarditis

An experienced nurse orients a new nurse to the postanesthesia care unit (PACU). Which action by the new nurse, if observed by the experienced nurse, indicates that the orientation was successful? a. The new nurse assists a nauseated patient to a supine position. b. The new nurse positions an unconscious patient supine with the head elevated. c. The new nurse turns an unconscious patient to the side upon arrival in the PACU. d. The new nurse places a patient in the Trendelenburg position when the blood pressure drops.

ANS: C The patient should initially be positioned in the lateral "recovery" position to keep the airway open and avoid aspiration. The Trendelenburg position is avoided because it increases the work of breathing. The patient is placed supine with the head elevated after regaining consciousness

An outpatient who has chronic heart failure returns to the clinic after 2 weeks of therapy with metoprolol (Toprol XL). Which assessment finding is most important for the nurse to report to the health care provider? a. 2+ pedal edema b. Heart rate of 56 beats/minute c. Blood pressure (BP) of 88/42 mm Hg d. Complaints of fatigue

ANS: C The patient's BP indicates that the dose of metoprolol may need to be decreased because of hypotension. Bradycardia is a frequent adverse effect of b-adrenergic blockade, but the rate of 56 is not unusual with â-adrenergic blocker therapy. b-Adrenergic blockade initially will worsen symptoms of heart failure in many patients, and patients should be taught that some increase in symptoms, such as fatigue and edema, is expected during the initiation of therapy with this class of drugs

A patient with possible disseminated intravascular coagulation arrives in the emergency department with a blood pressure of 82/40, temperature 102° F (38.9° C), and severe back pain. Which physician order will the nurse implement first? a. Administer morphine sulfate 4 mg IV. b. Give acetaminophen (Tylenol) 650 mg. c. Infuse normal saline 500 mL over 30 minutes. d. Schedule complete blood count and coagulation studies.

ANS: C The patient's blood pressure indicates hypovolemia caused by blood loss and should be addressed immediately to improve perfusion to vital organs. The other actions also are appropriate and should be rapidly implemented, but improving perfusion is the priority for this patient

A postoperative patient receiving a transfusion of packed red blood cells develops chills, fever, headache, and anxiety 35 minutes after the transfusion is started. After stopping the transfusion, what action should the nurse take? a. Draw blood for a new crossmatch. b. Send a urine specimen to the laboratory. c. Administer PRN acetaminophen (Tylenol). d. Give the PRN diphenhydramine (Benadryl).

ANS: C The patient's clinical manifestations are consistent with a febrile, nonhemolytic transfusion reaction. The transfusion should be stopped and antipyretics administered for the fever as ordered. A urine specimen is needed if an acute hemolytic reaction is suspected. Diphenhydramine (Benadryl) is used for allergic reactions. This type of reaction does not indicate incorrect crossmatching

The nurse reviews the medication administration record in order to choose the most appropriate pain medication for a patient with cancer who describes the pain as "deep, aching and at a level 8 on a 0 to 10 scale". Which medication should the nurse administer? a. Fentanyl (Duragesic) patch b. Ketorolac (Toradol) tablets c. Hydromorphone (Dilaudid) IV d. Acetaminophen (Tylenol) suppository

ANS: C The patient's pain level indicates that a rapidly acting medication such as an IV opioid is needed. The other medications also may be appropriate to use, but will not work as rapidly or as effectively as the IV hydromorphone

The nurse is caring for a patient who has diabetes and complains of chronic burning leg pain even when taking oxycodone (OxyContin) twice daily. When reviewing the orders, which prescribed medication is the best choice for the nurse to administer as an adjuvant to decrease the patient's pain? a. Aspirin (Ecotrin) b. Celecoxib (Celebrex) c. Amitriptyline (Elavil) d. Acetaminophen (Tylenol)

ANS: C The patient's pain symptoms are consistent with neuropathic pain and the tricyclic antidepressants are effective for treating this type of pain. The other medications are more effective for nociceptive pain

When caring for a patient with mitral valve stenosis, it is most important that the nurse assess for a. diastolic murmur. b. peripheral edema. c. shortness of breath on exertion. d. right upper quadrant tenderness.

ANS: C The pressure gradient changes in mitral stenosis lead to fluid backup into the lungs, resulting in hypoxemia and dyspnea. The other findings also may be associated with mitral valve disease but are not indicators of possible hypoxemia

The health care provider's progress note for a patient states that the complete blood count (CBC) shows a "shift to the left." Which assessment finding will the nurse expect? a. Cool extremities b. Pallor and weakness c. Elevated temperature d. Low oxygen saturation

ANS: C The term shift to the left indicates that the number of immature polymorphonuclear neutrophils (bands) is elevated and that finding is a sign of infection. There is no indication that the patient is at risk for hypoxemia, pallor/weakness, or cool extremities

A patient with a history of chronic heart failure is admitted to the emergency department (ED) with severe dyspnea and a dry, hacking cough. Which action should the nurse do first? a. Auscultate the abdomen. b. Check the capillary refill. c. Auscultate the breath sounds. d. Assess the level of orientation.

ANS: C This patient's severe dyspnea and cough indicate that acute decompensated heart failure (ADHF) is occurring. ADHF usually manifests as pulmonary edema, which should be detected and treated immediately to prevent ongoing hypoxemia and cardiac/respiratory arrest. The other assessments will provide useful data about the patient's volume status and also should be accomplished rapidly, but detection (and treatment) of pulmonary complications is the priority

Which nursing action can the registered nurse (RN) delegate to experienced unlicensed assistive personnel (UAP) working as a telemetry technician on the cardiac care unit? a. Decide whether a patient's heart rate of 116 requires urgent treatment. b. Monitor a patient's level of consciousness during synchronized cardioversion. c. Observe cardiac rhythms for multiple patients who have telemetry monitoring. d. Select the best lead for monitoring a patient admitted with acute coronary syndrome.

ANS: C UAP serving as telemetry technicians can monitor cardiac rhythms for individuals or groups of patients. Nursing actions such as assessment and choice of the most appropriate lead based on ST segment elevation location require RN-level education and scope of practice

The nurse assesses that a patient receiving epidural morphine has not voided for over 10 hours. What action should the nurse take initially? a. Monitor for withdrawal symptoms. b. Place an indwelling urinary catheter. c. Ask if the patient feels the need to void. d. Document this allergic reaction in the patient's chart.

ANS: C Urinary retention is a common side effect of epidural opioids. Assess whether the patient feels the need to void. Since urinary retention is a possible side effect, there is no reason for concern of withdrawal symptoms. Placing an indwelling catheter requires an order from the health care provider. Usually an in and out catheter is performed to empty the bladder if the patient is unable to void because of the risk of infection with an indwelling catheter. Urinary retention does not indicate that this reaction is an allergic reaction

A 56-year-old patient who has no previous history of hypertension or other health problems suddenly develops a blood pressure (BP) of 198/110 mm Hg. After reconfirming the BP, it is appropriate for the nurse to tell the patient that a. a BP recheck should be scheduled in a few weeks. b. dietary sodium and fat content should be decreased. c. there is an immediate danger of a stroke and hospitalization will be required. d. diagnosis of a possible cause, treatment, and ongoing monitoring will be needed.

ANS: D A sudden increase in BP in a patient over age 50 with no previous hypertension history or risk factors indicates that the hypertension may be secondary to some other problem. The BP will need treatment and ongoing monitoring. If the patient has no other risk factors, a stroke in the immediate future is unlikely. There is no indication that dietary salt or fat intake have contributed to this sudden increase in BP, and reducing intake of salt and fat alone will not be adequate to reduce this BP to an acceptable level

Following an acute myocardial infarction, a previously healthy 63-year-old develops clinical manifestations of heart failure. The nurse anticipates discharge teaching will include information about a. digitalis preparations. b. b-adrenergic blockers. c. calcium channel blockers. d. angiotensin-converting enzyme (ACE) inhibitors.

ANS: D ACE inhibitor therapy is currently recommended to prevent the development of heart failure in patients who have had a myocardial infarction and as a first-line therapy for patients with chronic heart failure. Digoxin therapy for heart failure is no longer considered a first-line measure, and digoxin is added to the treatment protocol when therapy with other medications such as ACE-inhibitors, diuretics, and b-adrenergic blockers is insufficient. Calcium channel blockers are not generally used in the treatment of heart failure. The b-adrenergic blockers are not used as initial therapy for new onset heart failure

The nurse is obtaining a health history from a 24-year-old patient with hypertrophic cardiomyopathy (HC). Which information obtained by the nurse is most important? a. The patient has a history of a recent upper respiratory infection. b. The patient has a family history of coronary artery disease (CAD). c. The patient reports using cocaine a "couple of times" as a teenager. d. The patient's 29-year-old brother died from a sudden cardiac arrest.

ANS: D About half of all cases of HC have a genetic basis, and it is the most common cause of sudden cardiac death in otherwise healthy young people. The information about the patient's brother will be helpful in planning care (such as an automatic implantable cardioverter-defibrillator [AICD]) for the patient and in counseling other family members. The patient should be counseled against the use of stimulant drugs, but the limited past history indicates that the patient is not at current risk for cocaine use. Viral infections and CAD are risk factors for dilated cardiomyopathy, but not for HC

A patient who has been receiving a heparin infusion and warfarin (Coumadin) for a deep vein thrombosis (DVT) is diagnosed with heparin-induced thrombocytopenia (HIT) when her platelet level drops to 110,000/µL. Which action will the nurse include in the plan of care? a. Use low-molecular-weight heparin (LMWH) only. b. Administer the warfarin (Coumadin) at the scheduled time. c. Teach the patient about the purpose of platelet transfusions. d. Discontinue heparin and flush intermittent IV lines using normal saline.

ANS: D All heparin is discontinued when the HIT is diagnosed. The patient should be instructed to never receive heparin or LMWH. Warfarin is usually not given until the platelet count has returned to 150,000/µL. The platelet count does not drop low enough in HIT for a platelet transfusion, and platelet transfusions increase the risk for thrombosis

A patient who has just been admitted with pulmonary edema is scheduled to receive the following medications. Which medication should the nurse question before giving? a. Furosemide (Lasix) 60 mg b. Captopril (Capoten) 25 mg c. Digoxin (Lanoxin) 0.125 mg d. Carvedilol (Coreg) 3.125 mg

ANS: D Although carvedilol is appropriate for the treatment of chronic heart failure, it is not used for patients with acute decompensated heart failure (ADHF) because of the risk of worsening the heart failure. The other medications are appropriate for the patient with ADHF

Which statement by a patient indicates good understanding of the nurse's teaching about prevention of sickle cell crisis? a. "Home oxygen therapy is frequently used to decrease sickling." b. "There are no effective medications that can help prevent sickling." c. "Routine continuous dosage narcotics are prescribed to prevent a crisis." d. "Risk for a crisis is decreased by having an annual influenza vaccination."

ANS: D Because infection is the most common cause of a sickle cell crisis, influenza, Haemophilus influenzae, pneumococcal pneumonia, and hepatitis immunizations should be administered. Although continuous dose opioids and oxygen may be administered during a crisis, patients do not receive these therapies to prevent crisis. Hydroxyurea (Hydrea) is a medication used to decrease the number of sickle cell crises

Which patient information is most important for the nurse to monitor when evaluating the effectiveness of deferoxamine (Desferal) for a patient with hemochromatosis? a. Skin color b. Hematocrit c. Liver function d. Serum iron level

ANS: D Because iron chelating agents are used to lower serum iron levels, the most useful information will be the patient's iron level. The other parameters will also be monitored, but are not the most important to monitor when determining the effectiveness of deferoxamine

When analyzing the rhythm of a patient's electrocardiogram (ECG), the nurse will need to investigate further upon finding a(n) a. isoelectric ST segment. b. P-R interval of 0.18 second. c. Q-T interval of 0.38 second. d. QRS interval of 0.14 second.

ANS: D Because the normal QRS interval is 0.04 to 0.10 seconds, the patient's QRS interval of 0.14 seconds indicates that the conduction through the ventricular conduction system is prolonged. The P-R interval and Q-T interval are within normal range, and ST segment should be isoelectric (flat).

The nurse is caring for a patient who is receiving IV furosemide (Lasix) and morphine for the treatment of acute decompensated heart failure (ADHF) with severe orthopnea. Which clinical finding is the best indicator that the treatment has been effective? a. Weight loss of 2 pounds in 24 hours b. Hourly urine output greater than 60 mL c. Reduction in patient complaints of chest pain d. Reduced dyspnea with the head of bed at 30 degrees

ANS: D Because the patient's major clinical manifestation of ADHF is orthopnea (caused by the presence of fluid in the alveoli), the best indicator that the medications are effective is a decrease in dyspnea with the head of the bed at 30 degrees. The other assessment data also may indicate that diuresis or improvement in cardiac output has occurred, but are not as specific to evaluating this patient's response

A nurse reviews the laboratory data for an older patient. The nurse would be most concerned about which finding? a. Hematocrit of 35% b. Hemoglobin of 11.8 g/dL c. Platelet count of 400,000/µL d. White blood cell (WBC) count of 2800/µL

ANS: D Because the total WBC count is not usually affected by aging, the low WBC count in this patient would indicate that the patient's immune function may be compromised and the underlying cause of the problem needs to be investigated. The platelet count is normal. The slight decrease in hemoglobin and hematocrit are not unusual for an older patient

The nurse is caring for a 70-year-old who uses hydrochlorothiazide (HydroDIURIL) and enalapril (Norvasc), but whose self-monitored blood pressure (BP) continues to be elevated. Which patient information may indicate a need for a change? a. Patient takes a daily multivitamin tablet. b. Patient checks BP daily just after getting up. c. Patient drinks wine three to four times a week. d. Patient uses ibuprofen (Motrin) daily to treat osteoarthritis.

ANS: D Because use of nonsteroidal antiinflammatory drugs (NSAIDs) can prevent adequate BP control, the patient may need to avoid the use of ibuprofen. A multivitamin tablet will help supply vitamin D, which may help lower BP. BP decreases while sleeping, so self-monitoring early in the morning will result in obtaining pressures that are at their lowest. The patient's alcohol intake is not excessive

The nurse obtains a health history from a 65-year-old patient with a prosthetic mitral valve who has symptoms of infective endocarditis (IE). Which question by the nurse is most appropriate? a. "Do you have a history of a heart attack?" b. "Is there a family history of endocarditis?" c. "Have you had any recent immunizations?" d. "Have you had dental work done recently?"

ANS: D Dental procedures place the patient with a prosthetic mitral valve at risk for infective endocarditis (IE). Myocardial infarction (MI), immunizations, and a family history of endocarditis are not risk factors for IE.

The health care provider orders a liver/spleen scan for a patient who has been in a motor vehicle accident. Which action should the nurse take before this procedure? a. Check for any iodine allergy. b. Insert a large-bore IV catheter. c. Place the patient on NPO status. d. Assist the patient to a flat position.

ANS: D During a liver/spleen scan, a radioactive isotope is injected IV and images from the radioactive emission are used to evaluate the structure of the spleen and liver. An indwelling IV catheter is not needed. The patient is placed in a flat position before the scan

When the nurse is monitoring a patient who is undergoing exercise (stress) testing on a treadmill, which assessment finding requires the most rapid action by the nurse? a. Patient complaint of feeling tired b. Pulse change from 87 to 101 beats/minute c. Blood pressure (BP) increase from 134/68 to 150/80 mm Hg d. Newly inverted T waves on the electrocardiogram

ANS: D ECG changes associated with coronary ischemia (such as T-wave inversions and ST segment depression) indicate that the myocardium is not getting adequate oxygen delivery and that the exercise test should be terminated immediately. Increases in BP and heart rate (HR) are normal responses to aerobic exercise. Feeling tired is also normal as the intensity of exercise increases during the stress testing

Which laboratory test will the nurse use to determine whether filgrastim (Neupogen) is effective for a patient with acute lymphocytic leukemia who is receiving chemotherapy? a. Platelet count b. Reticulocyte count c. Total lymphocyte count d. Absolute neutrophil count

ANS: D Filgrastim increases the neutrophil count and function in neutropenic patients. Although total lymphocyte, platelet, and reticulocyte counts also are important to monitor in this patient, the absolute neutrophil count is used to evaluate the effects of filgrastim

A patient has a normal cardiac rhythm and a heart rate of 72 beats/minute. The nurse determines that the P-R interval is 0.24 seconds. The most appropriate intervention by the nurse would be to a. notify the health care provider immediately. b. give atropine per agency dysrhythmia protocol. c. prepare the patient for temporary pacemaker insertion. d. document the finding and continue to monitor the patient.

ANS: D First-degree atrioventricular (AV) block is asymptomatic and requires ongoing monitoring because it may progress to more serious forms of heart block. The rate is normal, so there is no indication that atropine is needed. Immediate notification of the health care provider about an asymptomatic rhythm is not necessary

A patient who has fibromyalgia tells the nurse, "I feel depressed because I ache too much to play golf." The patient says the pain is usually at a level 7 (0 to 10 scale). Which patient goal has the highest priority when the nurse is developing the treatment plan? a. The patient will exhibit fewer signs of depression. b. The patient will say that the aching has decreased. c. The patient will state that pain is at a level 2 of 10. d. The patient will be able to play 1 to 2 rounds of golf.

ANS: D For chronic pain, patients are encouraged to set functional goals such as being able to perform daily activities and hobbies. The patient has identified playing golf as the desired activity, so a pain level of 2 of 10 or a decrease in aching would be less useful in evaluating successful treatment. The nurse should also assess for depression, but the patient has identified the depression as being due to the inability to play golf, so the goal of being able to play 1 or 2 rounds of golf is the most appropriate

After the nurse teaches the patient with stage 1 hypertension about diet modifications that should be implemented, which diet choice indicates that the teaching has been effective? a. The patient avoids eating nuts or nut butters. b. The patient restricts intake of chicken and fish. c. The patient has two cups of coffee in the morning. d. The patient has a glass of low-fat milk with each meal.

ANS: D For the prevention of hypertension, the Dietary Approaches to Stop Hypertension (DASH) recommendations include increasing the intake of calcium-rich foods. Caffeine restriction and decreased protein intake are not included in the recommendations. Nuts are high in beneficial nutrients and 4 to 5 servings weekly are recommended in the DASH diet.

Which action will be included in the plan of care when the nurse is caring for a patient who is receiving nicardipine (Cardene) to treat a hypertensive emergency? a. Keep the patient NPO to prevent aspiration caused by nausea and possible vomiting. b. Organize nursing activities so that the patient has undisturbed sleep for 6 to 8 hours at night. c. Assist the patient up in the chair for meals to avoid complications associated with immobility. d. Use an automated noninvasive blood pressure machine to obtain frequent blood pressure (BP) measurements.

ANS: D Frequent monitoring of BP is needed when the patient is receiving rapid-acting IV antihypertensive medications. This can be most easily accomplished with an automated BP machine or arterial line. The patient will require frequent assessments, so allowing 6 to 8 hours of undisturbed sleep is not appropriate. When patients are receiving IV vasodilators, bed rest is maintained to prevent decreased cerebral perfusion and fainting. There is no indication that this patient is nauseated or at risk for aspiration, so an NPO status is unnecessary

Which information should the nurse include when teaching a patient with newly diagnosed hypertension? a. Increasing physical activity will control blood pressure (BP) for most patients. b. Most patients are able to control BP through dietary changes. c. Annual BP checks are needed to monitor treatment effectiveness. d. Hypertension is usually asymptomatic until target organ damage occurs.

ANS: D Hypertension is usually asymptomatic until target organ damage has occurred. Lifestyle changes (e.g., physical activity, dietary changes) are used to help manage blood pressure, but drugs are needed for most patients. Home BP monitoring should be taught to the patient and findings checked by the health care provider frequently when starting treatment for hypertension and then every 3 months once stable

Which laboratory result for a patient with multifocal premature ventricular contractions (PVCs) is most important for the nurse to communicate to the health care provider? a. Blood glucose 243 mg/dL b. Serum chloride 92 mEq/L c. Serum sodium 134 mEq/L d. Serum potassium 2.9 mEq/L

ANS: D Hypokalemia increases the risk for ventricular dysrhythmias such as PVCs, ventricular tachycardia, and ventricular fibrillation. The health care provider will need to prescribe a potassium infusion to correct this abnormality. Although the other laboratory values also are abnormal, they are not likely to be the etiology of the patient's PVCs and do not require immediate correction

A nurse assesses a patient with chronic cancer pain who is receiving imipramine (Tofranil) in addition to long-acting morphine. Which statement, if made by the patient, indicates to the nurse that the patient is receiving adequate pain control? a. "I'm not anxious at all." b. "I sleep 8 hours every night." c. "I feel much less depressed since I've been taking the Tofranil." d. "The pain is manageable and I can accomplish my desired activities.

ANS: D Imipramine is being used in this patient to manage chronic pain and improve functional ability. Although the medication is also prescribed for patients with depression, insomnia, and anxiety, the evaluation for this patient is based on improved pain control and activity level

To determine the effects of therapy for a patient who is being treated for heart failure, which laboratory result will the nurse plan to review? a. Troponin b. Homocysteine (Hcy) c. Low-density lipoprotein (LDL) d. B-type natriuretic peptide (BNP)

ANS: D Increased levels of BNP are a marker for heart failure. The other laboratory results would be used to assess for myocardial infarction (troponin) or risk for coronary artery disease (Hcy and LDL).

A patient who has non-Hodgkin's lymphoma is receiving combination treatment with rituximab (Rituxan) and chemotherapy. Which patient assessment finding requires the most rapid action by the nurse? a. Anorexia b. Vomiting c. Oral ulcers d. Lip swelling

ANS: D Lip swelling in angioedema may indicate a hypersensitivity reaction to the rituximab. The nurse should stop the infusion and further assess for anaphylaxis. The other findings may occur with chemotherapy, but are not immediately life threatening

The nurse is completing the medication reconciliation form for a patient admitted with chronic cancer pain. Which medication is of most concern to the nurse? a. Amitriptyline (Elavil) 50 mg at bedtime b. Ibuprofen (Advil) 800 mg 3 times daily c. Oxycodone (OxyContin) 80 mg twice daily d. Meperidine (Demerol) 25 mg every 4 hours

ANS: D Meperidine is contraindicated for chronic pain because it forms a metabolite that is neurotoxic and can cause seizures when used for prolonged periods. The ibuprofen, amitriptyline, and oxycodone are all appropriate medications for long-term pain management

When teaching the patient with newly diagnosed heart failure about a 2000-mg sodium diet, the nurse explains that foods to be restricted include a. canned and frozen fruits. b. fresh or frozen vegetables. c. eggs and other high-protein foods. d. milk, yogurt, and other milk products.

ANS: D Milk and yogurt naturally contain a significant amount of sodium, and intake of these should be limited for patients on a diet that limits sodium to 2000 mg daily. Other milk products, such as processed cheeses, have very high levels of sodium and are not appropriate for a 2000-mg sodium diet. The other foods listed have minimal levels of sodium and can be eaten without restriction

A critical action by the nurse caring for a patient with an acute exacerbation of polycythemia vera is to a. place the patient on bed rest. b. administer iron supplements. c. avoid use of aspirin products. d. monitor fluid intake and output.

ANS: D Monitoring hydration status is important during an acute exacerbation because the patient is at risk for fluid overload or underhydration. Aspirin therapy is used to decrease risk for thrombosis. The patient should be encouraged to ambulate to prevent deep vein thrombosis (DVT). Iron is contraindicated in patients with polycythemia vera

A patient who uses a fentanyl (Duragesic) patch for chronic cancer pain suddenly complains of rapid onset pain at a level 9 (0 to 10 scale) and requests "something for pain that will work now." How will the nurse document the type of pain reported by this patient? a. Somatic pain b. Referred pain c. Neuropathic pain d. Breakthrough pain

ANS: D Pain that occurs beyond the chronic pain already being treated by appropriate analgesics is termed breakthrough pain. Neuropathic pain is caused by damage to peripheral nerves or the central nervous system (CNS). Somatic pain is localized and arises from bone, joint, muscle, skin, or connective tissue. Referred pain is pain that is localized in uninjured tissue

When assessing a newly admitted patient, the nurse notes pallor of the skin and nail beds. The nurse should ensure that which laboratory test has been ordered? a. Platelet count b. Neutrophil count c. White blood cell count d. Hemoglobin (Hgb) level

ANS: D Pallor of the skin or nail beds is indicative of anemia, which would be indicated by a low Hgb level. Platelet counts indicate a person's clotting ability. A neutrophil is a type of white blood cell that helps to fight infection

The nurse establishes the nursing diagnosis of ineffective health maintenance related to lack of knowledge regarding long-term management of rheumatic fever when a 30-year-old recovering from rheumatic fever without carditis says which of the following? a. "I will need prophylactic antibiotic therapy for 5 years." b. "I will need to take aspirin or ibuprofen (Motrin) to relieve my joint pain." c. "I will call the doctor if I develop excessive fatigue or difficulty breathing." d. "I will be immune to further episodes of rheumatic fever after this infection."

ANS: D Patients with a history of rheumatic fever are more susceptible to a second episode. Patients with rheumatic fever without carditis require prophylaxis until age 20 and for a minimum of 5 years. The other patient statements are correct and would not support the nursing diagnosis of ineffective health maintenance

Which laboratory result will the nurse expect to show a decreased value if a patient develops heparin-induced thrombocytopenia (HIT)? a. Prothrombin time b. Erythrocyte count c. Fibrinogen degradation products d. Activated partial thromboplastin time

ANS: D Platelet aggregation in HIT causes neutralization of heparin, so that the activated partial thromboplastin time will be shorter and more heparin will be needed to maintain therapeutic levels. The other data will not be affected by HIT

After noting a pulse deficit when assessing a 74-year-old patient who has just arrived in the emergency department, the nurse will anticipate that the patient may require a. emergent cardioversion. b. a cardiac catheterization. c. hourly blood pressure (BP) checks. d. electrocardiographic (ECG) monitoring.

ANS: D Pulse deficit is a difference between simultaneously obtained apical and radial pulses. It indicates that there may be a cardiac dysrhythmia that would best be detected with ECG monitoring. Frequent BP monitoring, cardiac catheterization, and emergent cardioversion are used for diagnosis and/or treatment of cardiovascular disorders but would not be as helpful in determining the immediate reason for the pulse deficit

The nurse has identified a nursing diagnosis of acute pain related to inflammatory process for a patient with acute pericarditis. The priority intervention by the nurse for this problem is to a. teach the patient to take deep, slow breaths to control the pain. b. force fluids to 3000 mL/day to decrease fever and inflammation. c. remind the patient to request opioid pain medication every 4 hours. d. place the patient in Fowler's position, leaning forward on the overbed table.

ANS: D Sitting upright and leaning forward frequently will decrease the pain associated with pericarditis. Forcing fluids will not decrease the inflammation or pain. Taking deep breaths will tend to increase pericardial pain. Opioids are not very effective at controlling pain caused by acute inflammatory conditions and are usually ordered PRN. The patient would receive scheduled doses of a nonsteroidal antiinflammatory drug (NSAID).

The nurse is caring for a patient who is being discharged after an emergency splenectomy following an automobile accident. Which instructions should the nurse include in the discharge teaching? a. Watch for excess bruising. b. Check for swollen lymph nodes. c. Take iron supplements to prevent anemia. d. Wash hands and avoid persons who are ill.

ANS: D Splenectomy increases the risk for infection, especially with gram-positive bacteria. The risks for lymphedema, bleeding, and anemia are not increased after a person has a splenectomy

During a physical examination of a 74-year-old patient, the nurse palpates the point of maximal impulse (PMI) in the sixth intercostal space lateral to the left midclavicular line. The most appropriate action for the nurse to take next will be to a. ask the patient about risk factors for atherosclerosis. b. document that the PMI is in the normal anatomic location. c. auscultate both the carotid arteries for the presence of a bruit. d. assess the patient for symptoms of left ventricular hypertrophy.

ANS: D The PMI should be felt at the intersection of the fifth intercostal space and the left midclavicular line. A PMI located outside these landmarks indicates possible cardiac enlargement, such as with left ventricular hypertrophy. Cardiac enlargement is not necessarily associated with atherosclerosis or carotid artery disease

The nurse obtains a rhythm strip on a patient who has had a myocardial infarction and makes the following analysis: no visible P waves, P-R interval not measurable, ventricular rate 162, R-R interval regular, and QRS complex wide and distorted, QRS duration 0.18 second. The nurse interprets the patient's cardiac rhythm as a. atrial flutter. b. sinus tachycardia. c. ventricular fibrillation. d. ventricular tachycardia.

ANS: D The absence of P waves, wide QRS, rate >150 beats/minute, and the regularity of the rhythm indicate ventricular tachycardia. Atrial flutter is usually regular, has a narrow QRS configuration, and has flutter waves present representing atrial activity. Sinus tachycardia has P waves. Ventricular fibrillation is irregular and does not have a consistent QRS duration

A patient has just been diagnosed with hypertension and has been started on captopril (Capoten). Which information is important to include when teaching the patient about this medication? a. Check blood pressure (BP) in both arms before taking the medication. b. Increase fluid intake if dryness of the mouth is a problem. c. Include high-potassium foods such as bananas in the diet. d. Change position slowly to help prevent dizziness and falls.

ANS: D The angiotensin-converting enzyme (ACE) inhibitors frequently cause orthostatic hypotension, and patients should be taught to change position slowly to allow the vascular system time to compensate for the position change. Increasing fluid intake may counteract the effect of the medication, and the patient is taught to use gum or hard candy to relieve dry mouth. The BP should be taken in the nondominant arm by newly diagnosed patients in the morning, before taking the medication, and in the evening. Because ACE inhibitors cause potassium retention, increased intake of high-potassium foods is inappropriate

A patient who was admitted with a myocardial infarction experiences a 45-second episode of ventricular tachycardia, then converts to sinus rhythm with a heart rate of 98 beats/minute. Which of the following actions should the nurse take next? a. Immediately notify the health care provider. b. Document the rhythm and continue to monitor the patient. c. Perform synchronized cardioversion per agency dysrhythmia protocol. d. Prepare to give IV amiodarone (Cordarone) per agency dysrhythmia protocol.

ANS: D The burst of sustained ventricular tachycardia indicates that the patient has significant ventricular irritability, and antidysrhythmic medication administration is needed to prevent further episodes. The nurse should notify the health care provider after the medication is started. Defibrillation is not indicated given that the patient is currently in a sinus rhythm. Documentation and continued monitoring are not adequate responses to this situation

Following successful treatment of Hodgkin's lymphoma for a 55-year-old woman, which topic will the nurse include in patient teaching? a. Potential impact of chemotherapy treatment on fertility b. Application of soothing lotions to treat residual pruritus c. Use of maintenance chemotherapy to maintain remission d. Need for follow-up appointments to screen for malignancy

ANS: D The chemotherapy used in treating Hodgkin's lymphoma results in a high incidence of secondary malignancies; follow-up screening is needed. The fertility of a 55-year-old woman will not be impacted by chemotherapy. Maintenance chemotherapy is not used for Hodgkin's lymphoma. Pruritus is a clinical manifestation of lymphoma, but should not be a concern after treatment

The nurse reviews the complete blood count (CBC) and white blood cell (WBC) differential of a patient admitted with abdominal pain. Which information will be most important for the nurse to communicate to the health care provider? a. Monocytes 4% b. Hemoglobin 13.6 g/dL c. Platelet count 168,000/µL d. White blood cells (WBCs) 15,500/µL

ANS: D The elevation in WBCs indicates that the patient has an inflammatory or infectious process ongoing, which may be the cause of the patient's pain, and that further diagnostic testing is needed. The monocytes are at a normal level. The hemoglobin and platelet counts are normal

The nurse has received change-of-shift report about the following patients on the progressive care unit. Which patient should the nurse see first? a. A patient who is in a sinus rhythm, rate 98, after having electrical cardioversion 2 hours ago b. A patient with new onset atrial fibrillation, rate 88, who has a first dose of warfarin (Coumadin) due c. A patient with second-degree atrioventricular (AV) block, type 1, rate 60, who is dizzy when ambulating d. A patient whose implantable cardioverter-defibrillator (ICD) fired two times today who has a dose of amiodarone (Cordarone) due

ANS: D The frequent firing of the ICD indicates that the patient's ventricles are very irritable, and the priority is to assess the patient and administer the amiodarone. The other patients may be seen after the amiodarone is administered

A patient with chronic neck pain is seen in the pain clinic for follow-up. In order to evaluate whether the pain management is effective, which question is best for the nurse to ask? a. "Can you describe the quality of your pain?" b. "Has there been a change in the pain location?" c. "How would you rate your pain on a 0 to 10 scale?" d. "Does the pain keep you from doing things you enjoy?"

ANS: D The goal for the treatment of chronic pain usually is to enhance function and quality of life. The other questions are also appropriate to ask, but information about patient function is more useful in evaluating effectiveness

A patient's cardiac monitor shows sinus rhythm, rate 64. The P-R interval is 0.18 seconds at 1:00 AM, 0.22 seconds at 2:30 PM, and 0.28 seconds at 4:00 PM. Which action should the nurse take next? a. Place the transcutaneous pacemaker pads on the patient. b. Administer atropine sulfate 1 mg IV per agency dysrhythmia protocol. c. Document the patient's rhythm and assess the patient's response to the rhythm. d. Call the health care provider before giving the next dose of metoprolol (Lopressor).

ANS: D The patient has progressive first-degree atrioventricular (AV) block, and the b-blocker should be held until discussing the medication with the health care provider. Documentation and assessment are appropriate but not fully adequate responses. The patient with first-degree AV block usually is asymptomatic, and a pacemaker is not indicated. Atropine is sometimes used for symptomatic bradycardia, but there is no indication that this patient is symptomatic

A patient develops sinus bradycardia at a rate of 32 beats/minute, has a blood pressure (BP) of 80/42 mm Hg, and is complaining of feeling faint. Which actions should the nurse take next? a. Recheck the heart rhythm and BP in 5 minutes. b. Have the patient perform the Valsalva maneuver. c. Give the scheduled dose of diltiazem (Cardizem). d. Apply the transcutaneous pacemaker (TCP) pads.

ANS: D The patient is experiencing symptomatic bradycardia, and treatment with TCP is appropriate. Continued monitoring of the rhythm and BP is an inadequate response. Calcium channel blockers will further decrease the heart rate, and the diltiazem should be held. The Valsalva maneuver will further decrease the rate

After providing a patient with discharge instructions on the management of a new permanent pacemaker, the nurse knows that teaching has been effective when the patient states a. "I will avoid cooking with a microwave oven or being near one in use." b. "It will be 1 month before I can take a bath or return to my usual activities." c. "I will notify the airlines when I make a reservation that I have a pacemaker." d. "I won't lift the arm on the pacemaker side up very high until I see the doctor."

ANS: D The patient is instructed to avoid lifting the arm on the pacemaker side above the shoulder to avoid displacing the pacemaker leads. The patient should notify airport security about the presence of a pacemaker before going through the metal detector, but there is no need to notify the airlines when making a reservation. Microwave oven use does not affect the pacemaker. The insertion procedure involves minor surgery that will have a short recovery period

The nurse teaches the patient being evaluated for rhythm disturbances with a Holter monitor to a. connect the recorder to a computer once daily. b. exercise more than usual while the monitor is in place. c. remove the electrodes when taking a shower or tub bath. d. keep a diary of daily activities while the monitor is worn.

ANS: D The patient is instructed to keep a diary describing daily activities while Holter monitoring is being accomplished to help correlate any rhythm disturbances with patient activities. Patients are taught that they should not take a shower or bath during Holter monitoring and that they should continue with their usual daily activities. The recorder stores the information about the patient's rhythm until the end of the testing, when it is removed and the data are analyzed

A patient admitted with acute dyspnea is newly diagnosed with dilated cardiomyopathy. Which information will the nurse plan to teach the patient about managing this disorder? a. A heart transplant should be scheduled as soon as possible. b. Elevating the legs above the heart will help relieve dyspnea. c. Careful compliance with diet and medications will prevent heart failure. d. Notify the doctor about any symptoms of heart failure such as shortness of breath.

ANS: D The patient should be instructed to notify the health care provider about any worsening of heart failure symptoms. Because dilated cardiomyopathy does not respond well to therapy, even patients with good compliance with therapy may have recurrent episodes of heart failure. Elevation of the legs above the heart will worsen symptoms (although this approach is appropriate for a patient with hypertrophic cardiomyopathy). The patient with terminal or end-stage cardiomyopathy may consider heart transplantation

A 62-year old man with chronic anemia is experiencing increased fatigue and occasional palpitations at rest. The nurse would expect the patient's laboratory findings to include a. a hematocrit (Hct) of 38%. b. an RBC count of 4,500,000/mL. c. normal red blood cell (RBC) indices. d. a hemoglobin (Hgb) of 8.6 g/dL (86 g/L).

ANS: D The patient's clinical manifestations indicate moderate anemia, which is consistent with a Hgb of 6 to 10 g/dL. The other values are all within the range of normal

A patient in the emergency department complains of back pain and difficulty breathing 15 minutes after a transfusion of packed red blood cells is started. The nurse's first action should be to a. administer oxygen therapy at a high flow rate. b. obtain a urine specimen to send to the laboratory. c. notify the health care provider about the symptoms. d. disconnect the transfusion and infuse normal saline.

ANS: D The patient's symptoms indicate a possible acute hemolytic reaction caused by the transfusion. The first action should be to disconnect the transfusion and infuse normal saline. The other actions also are needed but are not the highest priority

When reviewing the 12-lead electrocardiograph (ECG) for a healthy 79-year-old patient who is having an annual physical examination, what will be of most concern to the nurse? a. The PR interval is 0.21 seconds. b. The QRS duration is 0.13 seconds. c. There is a right bundle-branch block. d. The heart rate (HR) is 42 beats/minute.

ANS: D The resting HR does not change with aging, so the decrease in HR requires further investigation. Bundle-branch block and slight increases in PR interval or QRS duration are common in older individuals because of increases in conduction time through the AV node, bundle of His, and bundle branches

A patient who is receiving sustained-release morphine sulfate (MS Contin) every 12 hours for chronic pain experiences level 9 (0 to 10 scale) breakthrough pain and anxiety. Which action by the nurse is best? a. Provide amitriptyline (Elavil) 10 mg orally. b. Administer lorazepam (Ativan) 1 mg orally. c. Offer ibuprofen (Motrin) 400 to 800 mg orally. d. Give immediate-release morphine 30 mg orally.

ANS: D The severe breakthrough pain indicates that the initial therapy should be a rapidly acting opioid, such as the immediate-release morphine. Lorazepam and amitriptyline may be appropriate to use as adjuvant therapy, but they are not likely to block severe breakthrough pain. Use of antianxiety agents for pain control is inappropriate because this patient's anxiety is caused by the pain

The nurse needs to quickly estimate the heart rate for a patient with a regular heart rhythm. Which method will be best to use? a. Count the number of large squares in the R-R interval and divide by 300. b. Print a 1-minute electrocardiogram (ECG) strip and count the number of QRS complexes. c. Calculate the number of small squares between one QRS complex and the next and divide into 1500. d. Use the 3-second markers to count the number of QRS complexes in 6 seconds and multiply by 10.

ANS: D This is the quickest way to determine the ventricular rate for a patient with a regular rhythm. All the other methods are accurate, but take longer

Which patient with pain should the nurse assess first? a. Patient with postoperative pain who received morphine sulfate IV 15 minutes ago b. Patient with neuropathic pain who has a dose of hydrocodone (Lortab) scheduled now c. Patient who received hydromorphone (Dilaudid) 1 hour ago and currently has a sedation scale of 2 d. Patient who returned from the postanesthesia care unit 2 hours ago and has a respiratory rate of 10

ANS: D This patient's respiratory rate indicates possible respiratory depression. The risk for oversedation is greatest in the first 4 hours after transfer from the postanesthesia care unit. Patients should be reassessed 30 minutes after receiving IV opioids for pain. A scheduled oral mediation does not need to be administered exactly at the scheduled time. A sedation scale of 2 indicates adequate pain control from hydromorphone

A patient with pancytopenia has a bone marrow aspiration from the left posterior iliac crest. Which action would be important for the nurse to take after the procedure? a. Elevate the head of the bed to 45 degrees. b. Apply a sterile 2-inch gauze dressing to the site. c. Use a half-inch sterile gauze to pack the wound. d. Have the patient lie on the left side for 1 hour.

ANS: D To decrease the risk for bleeding, the patient should lie on the left side for 30 to 60 minutes. After a bone marrow biopsy, the wound is small and will not be packed with gauze. A pressure dressing is used to cover the aspiration site. There is no indication to elevate the patient's head

Which action could the nurse delegate to unlicensed assistive personnel (UAP) trained as electrocardiogram (ECG) technicians working on the cardiac unit? a. Select the best lead for monitoring a patient with an admission diagnosis of Dressler syndrome. b. Obtain a list of herbal medications used at home while admitting a new patient with pericarditis. c. Teach about the need to monitor the weight daily for a patient who has hypertrophic cardiomyopathy. d. Check the heart monitor for changes in rhythm while a patient who had a valve replacement ambulates.

ANS: D Under the supervision of registered nurses (RNs), UAP check the patient's cardiac monitor and obtain information about changes in heart rate and rhythm with exercise. Teaching and obtaining information about home medications (prescribed or complementary) and selecting the best leads for monitoring patients require more critical thinking and should be done by the RN

While caring for a 23-year-old patient with mitral valve prolapse (MVP) without valvular regurgitation, the nurse determines that discharge teaching has been effective when the patient states that it will be necessary to a. take antibiotics before any dental appointments. b. limit physical activity to avoid stressing the heart. c. take an aspirin a day to prevent clots from forming on the valve. d. avoid use of over-the-counter (OTC) medications that contain stimulant drugs.

ANS: D Use of stimulant medications should be avoided by patients with MVP because these may exacerbate symptoms. Daily aspirin and restricted physical activity are not needed by patients with mild MVP. Antibiotic prophylaxis is needed for patients with MVP with regurgitation but will not be necessary for this patient

While doing the admission assessment for a thin 76-year-old patient, the nurse observes pulsation of the abdominal aorta in the epigastric area. Which action should the nurse take? a. Teach the patient about aneurysms. b. Notify the hospital rapid response team. c. Instruct the patient to remain on bed rest. d. Document the finding in the patient chart.

ANS: D Visible pulsation of the abdominal aorta is commonly observed in the epigastric area for thin individuals. The nurse should simply document the finding in the admission assessment. Unless there are other abnormal findings (such as a bruit, pain, or hyper/hypotension) associated with the pulsation, the other actions are not necessary

While assessing a patient who was admitted with heart failure, the nurse notes that the patient has jugular venous distention (JVD) when lying flat in bed. Which action should the nurse take next? a. Document this finding in the patient's record. b. Obtain vital signs, including oxygen saturation. c. Have the patient perform the Valsalva maneuver. d. Observe for JVD with the patient upright at 45 degrees.

ANS: D When the patient is lying flat, the jugular veins are at the level of the right atrium, so JVD is a common (but not a clinically significant) finding. Obtaining vital signs and oxygen saturation is not warranted at this point. JVD is an expected finding when a patient performs the Valsalva maneuver because right atrial pressure increases. JVD that persists when the patient is sitting at a 30- to 45-degree angle or greater is significant. The nurse will document the JVD in the medical record if it persists when the head is elevated

The nurse assesses a patient on the second postoperative day after abdominal surgery to repair a perforated duodenal ulcer. Which finding is most important for the nurse to report to the surgeon? a. Tympanic temperature 99.2° F (37.3° C) b. Fine crackles audible at both lung bases c. Redness and swelling along the suture line d. 200 mL sanguineous fluid in the wound drain

ANS: D Wound drainage should decrease and change in color from sanguineous to serosanguineous by the second postoperative day. The color and amount of drainage for this patient are abnormal and should be reported. Redness and swelling along the suture line and a slightly elevated temperature are normal signs of postoperative inflammation. Atelectasis is common after surgery. The nurse should have the patient cough and deep breathe, but there is no urgent need to notify the surgeon

After teaching a patient with chronic stable angina about nitroglycerin, the nurse recognizes the need for further teaching when the patient makes which statement? A "I will replace my nitroglycerin supply every 6 months." B "I can take up to five tablets every 3 minutes for relief of my chest pain." C "I will take acetaminophen (Tylenol) to treat the headache caused by nitroglycerin." D "I will take the nitroglycerin 10 minutes before planned activity that usually causes chest pain."

B "I can take up to five tablets every 3 minutes for relief of my chest pain." The recommended dose of nitroglycerin is one tablet taken sublingually (SL) or one metered spray for symptoms of angina. If symptoms are unchanged or worse after 5 minutes, the patient should be instructed to activate the emergency medical services (EMS) system. If symptoms are improved, repeat the nitroglycerin every 5 minutes for a maximum of three doses and contact EMS if symptoms have not resolved completely.

Which patient is at greatest risk for sudden cardiac death? A A 42-year-old white woman with hypertension and dyslipidemia B A 52-year-old African American man with left ventricular failure C A 62-year-old obese man with diabetes mellitus and high cholesterol D A 72-year-old Native American woman with a family history of heart disease

B A 52-year-old African American man with left ventricular failure Patients with left ventricular dysfunction (ejection fraction < 30%) and ventricular dysrhythmias after MI are at greatest risk for sudden cardiac death (SCD). Other risk factors for SCD include (1) male gender (especially African American men), (2) family history of premature atherosclerosis, (3) tobacco use, (4) diabetes mellitus, (5) hypercholesterolemia, (6) hypertension, and (7) cardiomyopathy.

Which individuals would the nurse identify as having the highest risk for CAD? A A 45-year-old depressed male with a high-stress job B A 60-year-old male with below normal homocysteine levels C A 54-year-old female vegetarian with elevated high-density lipoprotein (HDL) levels D A 62-year-old female who has a sedentary lifestyle and body mass index (BMI) of 23 kg/m2

B A 60-year-old male with below normal homocysteine levels The 45-year-old depressed male with a high-stress job is at the highest risk for CAD. Studies demonstrate that depression and stressful states can contribute to the development of CAD. Elevated HDL levels and low homocysteine levels actually help to prevent CAD. Although a sedentary lifestyle is a risk factor, a BMI of 23 kg/m2 depicts normal weight, and thus the patient with two risk factors is at greatest risk for developing CAD.

The patient comes to the ED with severe, prolonged angina that is not immediately reversible. The nurse knows that if the patient once had angina related to a stable atherosclerotic plaque and the plaque ruptures, there may be occlusion of a coronary vessel and this type of pain. How will the nurse document this situation related to pathophysiology, presentation, diagnosis, prognosis, and interventions for this disorder? A Unstable angina B Acute coronary syndrome (ACS) C ST-segment-elevation myocardial infarction (STEMI) D Non-ST-segment-elevation myocardial infarction (NSTEMI)

B Acute coronary syndrome (ACS) The pain with ACS is severe, prolonged, and not easy to relieve. ACS is associated with deterioration of a once-stable atherosclerotic plaque that ruptures, exposes the intima to blood, and stimulates platelet aggregation and local vasoconstriction with thrombus formation. The unstable lesion, if partially occlusive, will be manifest as unstable angina or NSTEMI. If there is total occlusion, it is manifest as a STEMI.

The nurse would assess a patient with complaints of chest pain for which clinical manifestations associated with a myocardial infarction (MI) (select all that apply)? A Flushing B Ashen skin C Diaphoresis D Nausea and vomiting E S3 or S4 heart sounds

B Ashen skin C Diaphoresis D Nausea and vomiting E S3 or S4 heart sounds During the initial phase of an MI, catecholamines are released from the ischemic myocardial cells, causing increased sympathetic nervous system (SNS) stimulation. This results in the release of glycogen, diaphoresis, and vasoconstriction of peripheral blood vessels. The patient's skin may be ashen, cool, and clammy (not flushed) as a result of this response. Nausea and vomiting may result from reflex stimulation of the vomiting center by severe pain. Ventricular dysfunction resulting from the MI may lead to the presence of the abnormal S3 and S4 heart sounds.

The nurse is providing teaching to a patient recovering from an MI. How should resumption of sexual activity be discussed? A Delegated to the primary care provider B Discussed along with other physical activities C Avoided because it is embarrassing to the patient D Accomplished by providing the patient with written material

B Discussed along with other physical activities Although some nurses may not feel comfortable discussing sexual activity with patients, it is a necessary component of patient teaching. It is helpful to consider sex as a physical activity and to discuss or explore feelings in this area when other physical activities are discussed. Although providing the patient with written material is appropriate, it should not replace a verbal dialogue that can address the individual patient's questions and concerns.

The nurse prepares a discharge teaching plan for a 44-year-old male patient who has recently been diagnosed with coronary artery disease. Which risk factor should the nurse plan to focus on during the teaching session? A Type A personality B Elevated serum lipids C Family cardiac history D Hyperhomocysteinemia

B Elevated serum lipids Dyslipidemia is one of the four major modifiable risk factors for coronary artery disease (CAD). The other major modifiable risk factors are hypertension, tobacco use, and physical inactivity. Research findings related to psychologic states (i.e., type A personality) as a risk factor for coronary artery disease have been inconsistent. Family history is a nonmodifiable risk factor. High homocysteine levels have been linked to an increased risk for CAD.

Which antilipemic medications should the nurse question for a patient with cirrhosis of the liver (select all that apply)? A Niacin (Nicobid) B Ezetimibe (Zetia) C Gemfibrozil (Lopid) D Atorvastatin (Lipitor) E Cholestyramine (Questran)

B Ezetimibe (Zetia) D Atorvastatin (Lipitor) Ezetimibe (Zetia) should not be used by patients with liver impairment. Adverse effects of atorvastatin (Lipitor), a statin drug, include liver damage and myopathy. Liver enzymes must be monitored frequently and the medication stopped if these enzymes increase. Niacin's side effects subside with time, although decreased liver function may occur with high doses. Cholestyramine is safe for long-term use.

The nurse is examining the ECG of a patient who has just been admitted with a suspected MI. Which ECG change is most indicative of prolonged or complete coronary occlusion? A Sinus tachycardia B Pathologic Q wave C Fibrillatory P waves D Prolonged PR interval

B Pathologic Q wave The presence of a pathologic Q wave, as often accompanies STEMI, is indicative of complete coronary occlusion. Sinus tachycardia, fibrillatory P waves (e.g., atrial fibrillation), or a prolonged PR interval (first-degree heart block) are not direct indicators of extensive occlusion.

A patient with varicose veins has been prescribed compression stockings. How should the nurse teach the patient to use these? A. "Try to keep your stockings on 24 hours a day, as much as possible." B. "While you're still lying in bed in the morning, put on your stockings." C. "Dangle your feet at your bedside for 5 minutes before putting on your stockings." D. "Your stockings will be most effective if you can remove them for a few minutes several times a day."

B. "While you're still lying in bed in the morning, put on your stockings." The patient with varicose veins should apply stockings in bed, before rising in the morning. Stockings should not be worn continuously, but they should not be removed several times daily. Dangling at the bedside prior to application is likely to decrease their effectiveness.

A nurse is caring for a patient with a diagnosis of deep venous thrombosis (DVT). The patient has an order to receive 30 mg enoxaparin (Lovenox). Which injection site should the nurse use to administer this medication safely? A. Buttock, upper outer quadrant B. Abdomen, anterior-lateral aspect C. Back of the arm, 2 inches away from a mole D. Anterolateral thigh, with no scar tissue nearby

B. Abdomen, anterior-lateral aspect Enoxaparin (Lovenox) is a low-molecular-weight (LMW) heparin that is given as a deep subcutaneous injection in the right and left anterolateral abdomen. All subcutaneous injections should be given away from scars, lesions, or moles.

A female patient with critical limb ischemia has had peripheral artery bypass surgery to improve her circulation. What care should the nurse provide on postoperative day 1? A. Keep the patient on bed rest. B. Assist the patient with walking several times. C. Have the patient sit in the chair several times. D. Place the patient on her side with knees flexed.

B. Assist the patient with walking several times. To avoid blockage of the graft or stent, the patient should walk several times on postoperative day 1 and subsequent days. Having the patient's knees flexed for sitting in a chair or in bed increase the risk of venous thrombosis and may place stress on the suture lines.

A 73-year-old man with dementia has a venous ulcer related to chronic venous insufficiency. The nurse should provide education on which type of diet for this patient and his caregiver? A. Low-fat diet B. High-protein diet C. Calorie-restricted diet D. High-carbohydrate diet

B. High-protein diet A patient with a venous ulcer should have a balanced diet with adequate protein, calories, and micronutrients; this type of diet is essential for healing. Nutrients most important for healing include protein, vitamins A and C, and zinc. Foods high in protein (e.g., meat, beans, cheese, tofu), vitamin A (green leafy vegetables), vitamin C (citrus fruits, tomatoes, cantaloupe), and zinc (meat, seafood) must be provided. Restricting fat or calories is not helpful for wound healing or in patients of normal weight. For overweight individuals with no active venous ulcer, a weight-loss diet should be considered.

The nurse is caring for a newly admitted patient with vascular insufficiency. The patient has a new order for enoxaparin (Lovenox) 30 mg subcutaneously. What should the nurse do to correctly administer this medication? A. Spread the skin before inserting the needle. B. Leave the air bubble in the prefilled syringe. C. Use the back of the arm as the preferred site. D. Sit the patient at a 30-degree angle before administration.

B. Leave the air bubble in the prefilled syringe. The nurse should not expel the air bubble from the prefilled syringe because it should be injected to clear the needle of medication and avoid leaving medication in the needle track in the tissue.

A patient was just diagnosed with acute arterial ischemia in the left leg secondary to atrial fibrillation. Which early clinical manifestation must be reported to the physician immediately to save the patient's limb? A. Paralysis B. Paresthesia C. Crampiness D. Referred pain

B. Paresthesia The physician must be notified immediately if any of the six Ps of acute arterial ischemia occur to prevent ischemia from quickly progressing to tissue necrosis and gangrene. The six Ps are paresthesia, pain, pallor, pulselessness, and poikilothermia, with paralysis being a very late sign indicating the death of nerves to the extremity. Crampy leg sensation is more common with varicose veins. The pain is not referred.

67-year-old man with peripheral artery disease is seen in the primary care clinic. Which symptom reported by the patient would indicate to the nurse that the patient is experiencing intermittent claudication? A. Patient complains of chest pain with strenuous activity. B. Patient says muscle leg pain occurs with continued exercise. C. Patient has numbness and tingling of all his toes and both feet. D. Patient states the feet become red if he puts them in a dependent position.

B. Patient says muscle leg pain occurs with continued exercise. Intermittent claudication is an ischemic muscle ache or pain that is precipitated by a consistent level of exercise, resolves within 10 minutes or less with rest, and is reproducible. Angina is the term used to describe chest pain with exertion. Paresthesia is the term used to describe numbness or tingling in the toes or feet. Reactive hyperemia is the term used to describe redness of the foot; if the limb is in a dependent position the term is dependent rubor.

A 62-year-old Hispanic male patient with diabetes mellitus has been diagnosed with peripheral artery disease (PAD). The patient is a smoker and has a history of gout. What should the nurse focus her teaching on to prevent complications for this patient? A. Gender B. Smoking C. Ethnicity D. Co-morbidities

B. Smoking Smoking is the most significant factor for this patient. PAD is a marker of advanced systemic atherosclerosis. Therefore tobacco cessation is essential to reduce PAD progression, CVD events, and mortality. Diabetes mellitus and hyperuricemia are also risk factors. Being male or Hispanic are not risk factors for PAD.

In caring for the patient with angina, the patient said, "I walked to the bathroom. While I was having a bowel movement, I started having the worst chest pain ever, like before I was admitted. I called for a nurse, but the pain is gone now." What further assessment data should the nurse obtain from the patient? A "What precipitated the pain?" B "Has the pain changed this time?" C "In what areas did you feel this pain?" D "Rate the pain on a scale from 0 to 10, with 0 being no pain and 10 being the worst pain you can imagine."

C "In what areas did you feel this pain?" Using PQRST, the assessment data not volunteered by the patient is the radiation of pain, the area the patient felt the pain, and if it radiated. The precipitating event was going to the bathroom and having a bowel movement. The quality of the pain was "like before I was admitted," although a more specific description may be helpful. Severity of the pain was the "worst chest pain ever," although an actual number may be needed. Timing is supplied by the patient describing when the pain occurred and that he had previously had this pain.

Postoperative care of a patient undergoing coronary artery bypass graft (CABG) surgery includes monitoring for what common complication? A Dehydration B Paralytic ileus C Atrial dysrhythmias D Acute respiratory distress syndrome

C Atrial dysrhythmias Postoperative dysrhythmias, specifically atrial dysrhythmias, are common in the first 3 days following CABG surgery. Although the other complications could occur, they are not common complications.

The nurse assesses the right femoral artery puncture site as soon as the patient arrives after having a stent inserted into a coronary artery. The insertion site is not bleeding or discolored. What should the nurse do next to ensure the femoral artery is intact? A Palpate the insertion site for induration. B Assess peripheral pulses in the right leg. C Inspect the patient's right side and back. D Compare the color of the left and right legs.

C Inspect the patient's right side and back. The best method to determine that the right femoral artery is intact after inspection of the insertion site is to logroll the patient to inspect the right side and back for retroperitoneal bleeding. The artery can be leaking and blood is drawn into the tissues by gravity. The peripheral pulses, color, and sensation of the right leg will be assessed per agency protocol.

A 74-year-old man with a history of prostate cancer and hypertension is admitted to the emergency department with substernal chest pain. Which action will the nurse complete before administering sublingual nitroglycerin? A Administer morphine sulfate IV. B Auscultate heart and lung sounds. C Obtain a 12-lead electrocardiogram (ECG). D Assess for coronary artery disease risk factors.

C Obtain a 12-lead electrocardiogram (ECG). If a patient has chest pain, the nurse should institute the following measures: (1) administer supplemental oxygen and position the patient in upright position unless contraindicated, (2) assess vital signs, (3) obtain a 12-lead ECG, (4) provide prompt pain relief first with a nitrate followed by an opioid analgesic if needed, and (5) auscultate heart sounds. Obtaining a 12-lead ECG during chest pain aids in the diagnosis.

A postoperative patient asks the nurse why the physician ordered daily administration of enoxaparin (Lovenox). Which reply by the nurse is most appropriate? A. "This medication will help prevent breathing problems after surgery, such as pneumonia." B. "This medication will help lower your blood pressure to a safer level, which is very important after surgery." C. "This medication will help prevent blood clots from forming in your legs until your level of activity, such as walking, returns to normal." D. "This medication is a narcotic pain medication that will help take away any muscle aches caused by positioning on the operating room table."

C. "This medication will help prevent blood clots from forming in your legs until your level of activity, such as walking, returns to normal." Enoxaparin is an anticoagulant that is used to prevent DVTs postoperatively. All other explanations/options do not describe the action/purpose of enoxaparin.

The nurse is caring for a patient who has been receiving warfarin (Coumadin) and digoxin (Lanoxin) as treatment for atrial fibrillation. Because the warfarin has been discontinued before surgery, the nurse should diligently assess the patient for which complication early in the postoperative period until the medication is resumed? A. Decreased cardiac output B. Increased blood pressure C. Cerebral or pulmonary emboli D. Excessive bleeding from incision or IV sites

C. Cerebral or pulmonary emboli Warfarin is an anticoagulant that is used to prevent thrombi from forming on the walls of the atria during atrial fibrillation. Once the medication is terminated, thrombi could again form. If one or more thrombi detach from the atrial wall, they could travel as cerebral emboli from the left atrium or pulmonary emboli from the right atrium.

A 32-year-old female is prescribed diltiazem (Cardizem) for Raynaud's phenomenon. To evaluate the patient's expected response to this medication, what is most important for the nurse to assess? A. Improved skin turgor B. Decreased cardiac rate C. Improved finger perfusion D. Decreased mean arterial pressure

C. Improved finger perfusion Raynaud's phenomenon is an episodic vasospastic disorder of small cutaneous arteries, most frequently involving the fingers and toes. Diltiazem (Cardizem) is a calcium channel blocker that relaxes smooth muscles of the arterioles by blocking the influx of calcium into the cells, thus reducing the frequency and severity of vasospastic attacks. Perfusion to the fingertips is improved and vasospastic attacks reduced. Diltiazem may decrease heart rate and blood pressure, but that is not the purpose in Raynaud's phenomenon. Skin turgor is most often a reflection of hydration status.

The nurse would determine that a postoperative patient is not receiving the beneficial effects of enoxaparin (Lovenox) after noting what during a routine shift assessment? A. Generalized weakness and fatigue B. Crackles bilaterally in the lung bases C. Pain and swelling in lower extremity D. Abdominal pain with decreased bowel sounds

C. Pain and swelling in lower extremity Enoxaparin is a low-molecular-weight heparin used to prevent the development of deep vein thromboses (DVTs) in the postoperative period. Pain and swelling in the lower extremity can indicate development of DVT and therefore may signal ineffective medication therapy.

The nurse is caring for a preoperative patient who has an order for vitamin K by subcutaneous injection. The nurse should verify that which laboratory study is abnormal before administering the dose? A. Hematocrit (Hct) B. Hemoglobin (Hgb) C. Prothrombin time (PT) D. Partial thromboplastin time (PTT)

C. Prothrombin time (PT) Vitamin K counteracts hypoprothrombinemia and/or reverses the effects of warfarin (Coumadin) and thus decreases the risk of bleeding. High values for either the prothrombin time (PT) or the international normalized ratio (INR) demonstrates the need for this medication.

The nurse instructs a 68-year-old woman with hypercholesterolemia about natural lipid-lowering therapies. The nurse determines further teaching is necessary if the patient makes which statement? A "Omega-3 fatty acids are helpful in reducing triglyceride levels." B "I should check with my physician before I start taking any herbal products." C "Herbal products do not go through as extensive testing as prescription drugs do." D "I will take garlic instead of my prescription medication to reduce my cholesterol."

D "I will take garlic instead of my prescription medication to reduce my cholesterol." Current evidence does not support using garlic in the treatment of elevated cholesterol. Strong evidence supports the use of omega-3 fatty acids for reduction of triglyceride levels. Many herbal products are not standardized and effects are not predictable. Patients should consult with their health care provider before starting herbal or natural therapies.

The patient is being dismissed from the hospital after ACS and will be attending rehabilitation. What information does the patient need to be taught about the early recovery phase of rehabilitation? A Therapeutic lifestyle changes should become lifelong habits. B Physical activity is always started in the hospital and continued at home. C Attention will focus on management of chest pain, anxiety, dysrhythmias, and other complications. D Activity level is gradually increased under cardiac rehabilitation team supervision and with ECG monitoring.

D Activity level is gradually increased under cardiac rehabilitation team supervision and with ECG monitoring. In the early recovery phase after the patient is dismissed from the hospital, the activity level is gradually increased under supervision and with ECG monitoring. The late recovery phase includes therapeutic lifestyle changes that become lifelong habits. In the first phase of recovery, activity is dependent on the severity of the angina or MI, and attention is focused on the management of chest pain, anxiety, dysrhythmias, and other complications. With early recovery phase, the cardiac rehabilitation team may suggest that physical activity be initiated at home, but this is not always done.

For which problem is percutaneous coronary intervention (PCI) most clearly indicated? A Chronic stable angina B Left-sided heart failure C Coronary artery disease D Acute myocardial infarction

D Acute myocardial infarction PCI is indicated to restore coronary perfusion in cases of myocardial infarction. Chronic stable angina and CAD are normally treated with more conservative measures initially. PCI is not relevant to the pathophysiology of heart failure, such as left-sided heart failure.

When evaluating a patient's knowledge regarding a low-sodium, low-fat cardiac diet, the nurse recognizes additional teaching is needed when the patient selects which food choice? A Baked flounder B Angel food cake C Baked potato with margarine D Canned chicken noodle soup

D Canned chicken noodle soup Canned soups are very high in sodium content. Patients need to be taught to read food labels for sodium and fat content.

A patient was admitted to the emergency department (ED) 24 hours earlier with complaints of chest pain that were subsequently attributed to ST-segment-elevation myocardial infarction (STEMI). What complication of MI should the nurse anticipate? A Unstable angina B Cardiac tamponade C Sudden cardiac death D Cardiac dysrhythmias

D Cardiac dysrhythmias The most common complication after MI is dysrhythmias, which are present in 80% of patients. Unstable angina is considered a precursor to MI rather than a complication. Cardiac tamponade is a rare event, and sudden cardiac death is defined as an unexpected death from cardiac causes. Cardiac dysfunction in the period following an MI would not be characterized as sudden cardiac death.

A patient experienced sudden cardiac death (SCD) and survived. What should the nurse expect to be used as preventive treatment for the patient? A External pacemaker B An electrophysiologic study (EPS) C Medications to prevent dysrhythmias D Implantable cardioverter-defibrillator (ICD)

D Implantable cardioverter-defibrillator (ICD) An ICD is the most common approach to preventing recurrence of SCD. An external pacemaker may be used in the hospital but will not be used for the patient living daily life at home. An EPS may be done to determine if a recurrence is likely and determine the most effective medication treatment. Medications to prevent dysrhythmias are used but are not the best prevention of SCD.

A female patient who has type 1 diabetes mellitus has chronic stable angina that is controlled with rest. She states that over the past few months she has required increasing amounts of insulin. What goal should the nurse use to plan care that should help prevent cardiovascular disease progression? A Exercise almost every day. B Avoid saturated fat intake. C Limit calories to daily limit. D Keep Hgb A1C less than 7%.

D Keep Hgb A1C less than 7%. If the Hgb A1C is kept below 7%, this means that the patient has had good control of her blood glucose over the past 3 months. The patient indicates that increasing amounts of insulin are being required to control her blood glucose. This patient may not be adhering to the dietary guidelines or therapeutic regimen, so teaching about how to maintain diet, exercise, and medications to maintain stable blood glucose levels will be needed to achieve this goal.

A male patient who has coronary artery disease (CAD) has serum lipid values of LDL cholesterol 98 mg/dL and HDL cholesterol 47 mg/dL. What should the nurse include in the patient teaching? A Consume a diet low in fats. B Reduce total caloric intake. C Increase intake of olive oil. D The lipid levels are normal.

D The lipid levels are normal. For men, the recommended LDL is less than 100 mg/dL, and the recommended level for HDL is greater than 40mg/dL. His normal lipid levels should be included in the patient teaching and encourage him to continue taking care of himself. Assessing his need for teaching related to diet should also be done.

The nurse is caring for a patient with a recent history of deep vein thrombosis (DVT). The patient now needs to undergo surgery for appendicitis. The nurse is reviewing the laboratory results for this patient before administering an ordered dose of vitamin K. The nurse determines that the medication is both safe to give and is most needed when the international normalized ratio (INR) is which result? A. 1.0 B. 1.2 C. 1.6 D. 2.2

D. 2.2 Vitamin K is the antidote to warfarin (Coumadin), which the patient has most likely been taking before admission for treatment of DVT. Warfarin is an anticoagulant that impairs the ability of the blood to clot. Therefore it is necessary to give vitamin K before surgery to reduce the risk of hemorrhage. The largest value of the INR indicates the greatest impairment of clotting ability, making 2.2 the correct selection.

A 39-year-old woman with a history of smoking and oral contraceptive use is admitted with a venous thromboembolism (VTE) and prescribed unfractionated heparin. What laboratory test should the nurse review to evaluate the expected effect of the heparin? A. Platelet count B. Activated clotting time (ACT) C. International normalized ratio (INR) D. Activated partial thromboplastin time (APTT)

D. Activated partial thromboplastin time (APTT) Unfractionated heparin can be given by continuous IV for VTE treatment. When given IV, heparin requires frequent laboratory monitoring of clotting status as measured by activated partial thromboplastin time (aPTT). Platelet counts can decrease as an adverse reaction to heparin, but that is not the expected effect.

A male patient was admitted for a possible ruptured aortic aneurysm, but had no back pain. Ten minutes later his assessment includes the following: sinus tachycardia at 138, BP palpable at 65 mm Hg, increasing waist circumference, and no urine output. How should the nurse interpret this assessment about the patient's aneurysm? A. Tamponade will soon occur. B. The renal arteries are involved. C. Perfusion to the legs is impaired. D. He is bleeding into the abdomen.

D. He is bleeding into the abdomen. The lack of back pain indicates the patient is most likely exsanguinating into the abdominal space, and the bleeding is likely to continue without surgical repair. A blockade of the blood flow will not occur in the abdominal space as it would in the retroperitoneal space where surrounding anatomic structures may control the bleeding. The lack of urine output does not indicate renal artery involvement, but that the bleeding is occurring above the renal arteries, which decreases the blood flow to the kidneys. There is no assessment data indicating decreased perfusion to the legs.

The nurse is preparing to administer a scheduled dose of enoxaparin (Lovenox) 30 mg subcutaneously. What should the nurse do to administer this medication correctly? A. Remove the air bubble in the prefilled syringe. B. Aspirate before injection to prevent IV administration. C. Rub the injection site after administration to enhance absorption. D. Pinch the skin between the thumb and forefinger before inserting the needle.

D. Pinch the skin between the thumb and forefinger before inserting the needle. The nurse should gather together or "bunch up" the skin between the thumb and the forefinger before inserting the needle into the subcutaneous tissue. The nurse should not remove the air bubble in the prefilled syringe, aspirate, nor rub the site after injection.

What is a priority nursing intervention in the care of a patient with a diagnosis of chronic venous insufficiency (CVI)? A. Application of topical antibiotics to venous ulcers B. Maintaining the patient's legs in a dependent position C. Administration of oral and/or subcutaneous anticoagulants D. Teaching the patient the correct use of compression stockings

D. Teaching the patient the correct use of compression stockings CVI requires conscientious and consistent application of compression stockings. Anticoagulants are not necessarily indicated and antibiotics, if required, are typically oral or IV, not topical. The patient should avoid prolonged positioning with the limb in a dependent position.

The nurse assesses that the oxygen saturation is 89% in an unconscious patient who was transferred from surgery to the postanesthesia care unit (PACU) 15 minutes ago. Which action should the nurse take first? a. Elevate the patient's head. b. Suction the patient's mouth. c. Increase the oxygen flow rate. d. Perform the jaw-thrust maneuver.

In an unconscious postoperative patient, a likely cause of hypoxemia is airway obstruction by the tongue, and the first action is to clear the airway by maneuvers such as the jaw thrust or chin lift. Increasing the oxygen flow rate and suctioning are not helpful when the airway is obstructed by the tongue. Elevating the patient's head will not be effective in correcting the obstruction but may help with oxygenation after the patient is awake

When the patient is being examined for venous thromboembolism (VTE) in the calf, what diagnostic test should the nurse expect to teach the patient about first? ■ Duplex ultrasound ■ Contrast venography ■ Magnetic resonance venography ■ Computed tomography venography

■ Duplex ultrasound The duplex ultrasound is the most widely used test to diagnose VTE. Contrast venography is rarely used now. Magnetic resonance venography is less accurate for calf veins than pelvic and proximal veins. Computed tomography venography may be used but is invasive and much more expensive than the duplex ultrasound.

The patient has CVI and a venous ulcer. The unlicensed assistive personnel (UAP) decides to apply compression stockings because that is what these patients always have ordered. What assessment by the nurse would cause the application of compression stockings to harm the patient? ■ Rest pain ■ High blood pressure ■ Elevated blood sugar ■ Dry, itchy, flaky skin

■ Rest pain Rest pain occurs as peripheral artery disease (PAD) progresses and involves multiple arterial segments. Compression stockings should not be used on patients with PAD. Elevated blood glucose, possibly indicating uncontrolled diabetes mellitus, and hypertension may or may not indicate arterial problems. Dry, itchy, flaky skin indicates venous insufficiency. The RN should be the one to obtain the order and instruct the UAP to apply compression stockings if they are ordered.

The patient had aortic aneurysm repair. What priority nursing action will the nurse use to maintain graft patency? ■ Assess output for renal dysfunction. ■ Use IV fluids to maintain adequate BP. ■ Use oral antihypertensives to maintain cardiac output. ■ Maintain a low BP to prevent pressure on surgical site

■ Use IV fluids to maintain adequate BP. The priority is to maintain an adequate BP (determined by the surgeon) to maintain graft patency. A prolonged low BP may result in graft thrombosis, and hypertension may cause undue stress on arterial anastomoses resulting in leakage of blood or rupture at the suture lines, which is when IV antihypertensives may be used. Renal output will be assessed when the aneurysm repair is above the renal arteries to assess graft patency, not maintain it.

The patient reports tenderness when she touches her leg over a vein. The nurse assesses warmth and a palpable cord in the area. The nurse knows the patient needs treatment to prevent which sequelae? ■ Pulmonary embolism ■ Pulmonary hypertension ■ Post-thrombotic syndrome ■ Venous thromboembolism

■ Venous thromboembolism The clinical manifestations are characteristic of a superficial vein thrombosis. If untreated, the clot may extend to deeper veins, and venous thromboembolism may occur. Pulmonary embolism, pulmonary hypertension, and post-thrombotic syndrome are the sequelae of venous thromboembolism.


Kaugnay na mga set ng pag-aaral

Chapter 4 Test (True/False, Multiple Choice, Fill in the Blank)

View Set

AP Bio Tri 2 Exam Multiple Choice

View Set

Verbs - Gerunds and Infinitives - Level 1

View Set